Old Q&A – Health Maintenance

A 52-year-old woman comes to the general medicine clinic for an annual employee physical examination. She has hypothyroidism which is being treated with levothyroxine and she is currently perimenopausal. She feels well and has no symptoms of hypothyroidism at this time. She recently saw a television program about colon cancer and the advent of virtual colonoscopy and asks you about her risk for developing colon cancer. She has no family history of colon cancer or other malignancies. She denies abdominal pain, weight loss, melena, or hematochezia. Her vitals and physical examination are normal. The most appropriate recommendation for this patient is

  A. annual barium enema
  B. annual colonoscopy
  C. annual CT of the abdomen and pelvis
  D. annual digital rectal examination
  E. annual fecal occult blood testing and sigmoidoscopy every 3-5 years
Explanation:

The correct answer is E. The principal screening tests for detecting colorectal cancer in asymptomatic patients age 50 and older are the fecal occult blood testing (FOBT) and sigmoidoscopy. The American Cancer Society recommends annual digital rectal examination for all adults beginning at age 40, annual FOBT beginning at age 50, and sigmoidoscopy every 3-5 years beginning at age 50. The reported sensitivity and specificity of FOBT for detecting colorectal cancer in asymptomatic persons are 26-92% and 90-99%, respectively. Sigmoidoscopy screening in asymptomatic persons detects 1-4 cancers per 1,000 examinations and supplements the FOBT.

Annual barium enema (choice A) is not the best screening test for colorectal cancer in an asymptomatic patient. The estimated sensitivity and specificity of barium enema in detecting lesions within its reach are about 80-95% and 90%, respectively. This diagnostic test is recommended for patients that are symptomatic or have an increased risk factor for colorectal cancer such as in the setting of hereditary polyposis, hereditary non-polyposis colorectal cancer, and long standing ulcerative colitis.

Annual colonoscopy (choice B) is not the best screening test for colorectal cancer in an asymptomatic patient. About 95% of colorectal cancers are within reach of colonoscopy and the examination has an estimated 75-95% sensitivity in detecting lesions within its reach. Colonoscopy, which requires sedation and often involves the use of a hospital suite, is more expensive than other screening tests and has a higher risk of complications due to the procedure itself, secondary to anesthesia. This diagnostic test is recommended for patients that are symptomatic or have an increased risk factor for colorectal cancer such as in the setting of hereditary polyposis, hereditary non-polyposis colorectal cancer, and long standing ulcerative colitis.

Annual CT of the abdomen and pelvis (choice C) has no established role in the screening of colorectal cancer. A CT, however, is a useful imaging tool once the diagnosis of colorectal cancer is made because it provides important staging information. The principal screening tests for detecting colorectal cancer in asymptomatic patients age 50 and older are the fecal occult blood testing (FOBT) and sigmoidoscopy. The American Cancer Society recommends annual digital rectal examination for all adults beginning at age 40, annual FOBT beginning at age 50, and sigmoidoscopy every 3-5 years beginning at age 50.

Annual digital rectal examination (choice D) is often used as a preliminary screen for colon cancer and is recommended annually for patients age 40 and older. It is not the best recommendation on its own because it is of limited value as a screening test for colorectal cancer. The examining finger, which is only 7 to 8 cm long, has limited access even to the rectal mucosa, which is 11 cm in length. Fewer than 10% of colorectal cancers can be palpated by the examining finger and as a result, a negative digital rectal examination provides little reassurance that the patient is free of colorectal cancer. For this asymptomatic patient who is 52 years old, it is best to recommend an annual fecal occult blood testing and a sigmoidoscopy every 3-5 years.

http://www.usmlestep.com

http://health.groups.yahoo.com/group/usmlestep3

Visit us to get more STEP 3 Materials

 

 

A 51-year-old woman with chronic low back pain comes to the office for an annual physical examination. She has been a patient for a number of years but you have not seen her for two years. Her last menstrula period was 9 months ago and she is not sexually active. She had a cholecystectomy at age 43 and an ovarian cyst removal at age 23. Her family history is remarkable for a father who died of a myocardial infarction at age 59 and a mother who is alive and well. Her sister and her aunt both died of breast cancer at age 61. Her father and her grandfather both had diabetes. Physical examination is unremarkable. Concerning her risk for cervical cancer, the most appropriate management is

  A. bi-manual examinations annually
  B. colposcopy every one to three years
  C. no screening is necessary as this patient is postmenopausal
  D. no screening is necessary based upon her age
  E. Pap smear testing every one to three years
Explanation:

The correct answer is E. Cervical cancer has steadily declined as a cause of death in women over the past 40 years. This is due in large part to the massive screening effort started within the last 30 years, with the Pap smear at its core. The recommendations are for women after first intercourse, or after age 18, to have a Pap smear performed every one to three years. After 65, routine screening with Pap smears may possibly be discontinued if they were previously consistently normal.

Even best estimates show that bi-manual examinations annually (choice A) would reduce long-term mortality from ovarian or cervical CA by less that 0.0001%.

Colposcopy every one to three years (choice B) is incorrect. Colposcopy is reserved for women who have Pap smears that suggest a low-grade lesion that fails to resolve over time or demonstrates the presence of a high-grade lesion.

Postmenopausal women still have a risk of cervical cancer (choice C) because part of the etiology of cervical cancer is presumed to be independent of hormones (HPV infection).

The recommendations for cervical cancer screening currently have Pap smears being offered until age 65, therefore some screening is necessary in this patient (choice D).

 

 

While in the hospital examining a patient with congestive heart failure, you run into a 30-year-old colleague who has just suffered a needle stick injury from a patient with known hepatitis B. She is very concerned about contracting the disease, because she has never been vaccinated against it. She is generally very healthy, takes a daily multivitamin, and an oral contraceptive agent. She smokes a few cigarettes a day, but denies any alcohol use. Her family history is significant for coronary disease and pancreatic carcinoma. Based upon the available information, you would advise her to

  A. check her hepatitis antibody titer and await results
  B. begin antiretroviral therapy immediately
  C. get the hepatitis B immune globulin (HBIG)
  D. get the hepatitis B vaccine
  E. get the hepatitis B vaccine and hepatitis B immune globulin (HBIG)
Explanation:

The correct answer is E. Anyone who is exposed to blood from a high-risk patient with hepatitis B should receive both the hepatitis vaccine and the hepatitis immune globulin (HBIG) for optimal prophylaxis against acquiring the disease.

Checking a hepatitis B titer and awaiting the results (choice A) is unlikely to yield useful information in a patient without a prior history of hepatitis B or a vaccination against hepatitis B. Also, as stated above, since she was exposed to blood from a high-risk patient with hepatitis B, she should receive both the hepatitis vaccine and the hepatitis immune globulin (HBIG) for optimal prophylaxis against acquiring the disease.

Antiretroviral therapy (choice B) is not used as part of the first-line treatment of exposure to hepatitis B.

Getting the hepatitis B immune globulin alone (choice C) is not sufficient, since the immune globulin will not provide long-term protection against infection.

Obtaining the hepatitis B vaccine alone (choice D) is not sufficient, since it will not provide sufficient immediate protection against infection.

 

 

 

  A 42-year-old man comes to the emergency department because of chest pain that is very focal, just adjacent to the sternum at approximately the junction of the left 4th rib. It does not hurt to take a deep breath, but the pain is exacerbated with certain twisting movements. He denies fever, cough, sputum production, nausea, vomiting, diaphoresis, or shortness of breath. He tells you that he thinks that “it is most likely nothing,” but he is very concerned since his father died of a myocardial infarction at age 45 and he has not seen a physician since he was 14 years old. He has no significant past medical history and is on no medications. He is afebrile with normal vital signs. Oxygen saturation is 100% on room air. He has point tenderness to light palpation over the left 4th costochondral junction. Heart is regular with no murmurs, rubs, or gallops. Lungs are clear and his abdomen is benign. Extremities are normal. Laboratory studies show a leukocyte count of 8,100 mm3, hematocrit of 34%, creatinine of 0.7 mg/dL, blood urea nitrogen of 18 mg/dL, and cardiac enzymes and troponin are normal. Electrocardiogram is normal. Chest x-ray is normal. You explain that he most likely has costochondritis and “prescribe as needed” nonsteroidal antiinflammatory medications. He tells you that he would like to follow up with you in the clinic because he is concerned about his cardiovascular risk. You should tell him that when he comes to the clinic you plan to order

  A. no laboratory tests since his pain will most likely have resolved
  B. a periodic fasting cholesterol profile and yearly blood pressure checks
  C. a periodic fasting cholesterol profile, yearly blood pressure checks, and an annual chest x-ray
  D. a periodic fasting cholesterol profile, yearly blood pressure checks, and a coronary angiogram
  E. a periodic fasting cholesterol profile, yearly blood pressure checks, and sigmoidoscopy
Explanation:

The correct answer is B. Despite the fact that his initial clinical presentation was chest pain, he clearly had costochondritis that has now resolved. Although he may have significant coronary artery disease, (just as anyone could), there is no clinical evidence of that at this time. He does however have some cardiac risk factors, namely his sex and family history. It is the physician’s responsibility to try to modify the risk factors that can be treated such as hypercholesterolemia and high blood pressure. It is thus important to perform periodic fasting cholesterol and blood pressure checks. Infact, even if he had no family history or was a woman, it is still part of good health maintenance to monitor these. For these reasons, no laboratory tests are needed on follow up visits, since his pain will most likely have resolved (choice A) is incorrect.

Annual screening chest x-rays (choice C) for lung cancer screening or for any other reason in asymptomatic patients is not currently recommended. Early detection with low dose CT scanning in high-risk patients is currently under heavy investigation and shows much promise.

Although as mentioned earlier, anyone could have coronary artery disease, but nothing is indicating this in him right now. Thus a coronary arteriogram (choice D) would not be indicated at this time.

Sigmoidoscopy (choice E) is also used in screening and health maintenance, but not recommended until the age of 50.

 

 

A 46-year-old woman comes to the office for a periodic health maintenance examination. She says that she feels “great”, exercises daily, eats a low-fat diet, takes her calcium, and gets regular mammograms and pap smears. Her physical examination is entirely normal. You notice that she has become fidgety as you turn to leave the room. You ask her if there is anything bothering her and she tells you that she is concerned about her 15-year-old son, who is also your patient. She says that she has walked in on her son masturbating six times in the past 2 weeks, and she found a “heterosexual adult magazine” under his mattress. She is worried about this “behavior”. She says that he is doing well in school, plays on the varsity basketball team, and hangs out with friends on the weekends. The most appropriate response to this patient is:

  A. “From what you have told me, this seems like completely normal behavior for an adolescent boy.”
  B. “He will regret it later on in life; masturbation causes infertility.”
  C. “I wish I could help you, but it seems that you need to take him to a psychiatrist for evaluation.”
  D. “It sounds like there is more to this story than you are telling me; he must have been sexually abused as a child.”
  E. “You should make him an appointment to see me as soon as possible; I need to evaluate him for a paraphilia”
  F. “You should reprimand him the next time you walk in on him masturbating.”
Explanation:

The correct answer is A. Masturbation is completely normal. The mother should be told to respect her son’s privacy at this time and try not to make him feel like he is doing anything wrong. Many adolescent boys hide pornographic magazines in their rooms, and as long as this “behavior” does not involve children or anything illegal and is not causing functional impairment, there is no need for concern.

It is inappropriate to say, “He will regret it later on in life; masturbation causes infertility.” (choice B) because this statement is not true.

It is inappropriate to say, “I wish I could help you, but it seems that you need to take him to a psychiatrist for evaluation.” (choice C) because masturbation is a completely normal and does not require any evaluation.

It is inappropriate to say, “It sounds like there is more to this story than you are telling me; he must have been sexually abused as a child.” (choice D) because there is no indication that he was sexually abused. Masturbation is a completely normal and typically is unrelated to previous sexual abuse. If you are concerned that there is something she is not telling you, it would be better to say, “It sounds like there is more than you are telling me, maybe we should discuss it.”

It is inappropriate to say, “You should make him an appointment to see me as soon as possible; I need to evaluate him for a paraphilia” (choice E) because masturbation is completely normal behavior and there is no need to evaluate him for a paraphilia. It is very common for a teenage boy to have a “heterosexual adult magazine” hidden in his room; and since there is no functional impairment and it does not seem like he is involved in abnormal behavior (such as inanimate objects or prepubescent children), there does not seem to be any cause for concern.

You should not tell her to reprimand him the next time she walks in on him masturbating(choice F). Masturbation is completely normal. The mother should be told to respect her son’s privacy at this time and try not to make him feel like he is doing anything wrong.

 

 

A 19-year-old college student comes to the clinic “to get some acne medication.” He tells you that he has been using over-the-counter lotions with salicylic acid that have not been effective. He has a few open and closed comedones on his cheeks, chin, and forehead. After prescribing an appropriate therapy you look in his chart and notice that his immunization history is blank. You ask about previous immunizations and he tells you that he is up-to-date with his measles mumps rubella, tetanus, diphtheria, and polio vaccines. He remembers that he and his sister had the chicken pox virus when he was 7 years of age. He is generally very healthy, participates on the college wrestling team, and is pledging a fraternity. He admits that he drinks “quite a bit of alcohol” at fraternity “hazing events.” He does not have a steady girlfriend, but is sexually active with a “few different girls.” At this time you should

  A. do nothing
  B. order liver function tests
  C. order serologic testing for varicella-zoster virus
  D. recommend the hepatitis B virus vaccine
  E. send HIV serologic testing
Explanation:

The correct answer is D. The hepatitis B virus vaccine is recommended for all sexually active adolescents and college students. It is necessary to check vaccination histories on all college students to try to protect against vaccine-preventable diseases. Multiple sexual partners, injection drug use, and any other activities with exposures to blood or bodily fluids increase the risk of hepatitis B virus.

Since he is sexually active with multiple partners, he should be given the hepatitis B vaccine and should be offered HIV counseling and testing. It is inappropriate to do nothing (choice A).

Even though he admits to drinking a lot of alcohol, there is no indication to order liver function tests (choice B). Heavy drinking in an asymptomatic individual does not warrant any investigation.

Since he remembers having the chicken pox virus at age 7, there is no reason to doubt his history and to order serologic testing for varicella-zoster virus (choice C). If he does not recall if he had the virus, serologic testing is indicated, and if seronegative, he should be immunized.

HIV counseling and testing should be offered to sexually active adolescents and college students. However, consent must be obtained before testing and therefore sending HIV serologic testing (choice E) without first obtaining consent is incorrect.

 

 

A 73-year-old man who was diagnosed with prostate cancer 3 years ago comes to the office for a health maintenance examination. He tells you that the bone pain from the prostate cancer metastases is becoming unbearable and he is uncomfortable for most of the day. His other big concern is money. Up until 6 months ago he was receiving money from his son, but his son lost his job, and so the money has stopped coming in. He has been having trouble paying for his prescription medications and one of his friends told him about “pill splitting.” He now breaks each pill in half and takes one half one day and the other half the next day. This way, his prescription lasts twice as long as before. His medications, which he took an hour before this appointment, include buspirone, controlled release oxycodone, and sertraline. His blood pressure is 100/60 mm Hg, pulse is 50/min, and respirations are 9/min. Physical examination shows constricted pupils, but is otherwise unremarkable. You should advise the patient that:

  A. He should consider going into a nursing home where they will be able to monitor his medications
  B. He should have called you before he decided to begin the practice of “pill splitting”
  C. This “pill splitting” practice is a good way to save money on his prescriptions
  D. While “pill splitting” may be considered okay for some pills, it is not okay for other pills
  E. You are going to stop prescribing the oxycodone if he continues to split the pills
Explanation:

The correct answer is D. “Pill splitting” is a practice that is being encouraged by some health insurance companies as a money saving method, however there are some pills that can be split safely and others that definitely cannot be spit safely. Pills that are “scored”, meaning that there is an indented line down the middle of the pill going completely from one side of the pill to the other, can be broken almost evenly with a pill splitting device. Pills that are not scored should not be broken because there is no way to tell if you are actually getting half of the active ingredients, and pieces usually crumble off. Pills that are enteric coated should not be split because the enteric coating will not be on the split part of the pill, and they will lose the mechanism of enteric coating. Finally, pills that have a sustained/controlled/extended release formulation should not be broken because the mechanism that releases the medication into the system in a steady state throughout the day, is destroyed. The slow-release mechanism is ruined, and the patient may not get any medication, or may end up getting the entire dose at once. As in this case, this patient is taking sustained release oxycodone and is obviously getting too much of the active ingredient too fast because he has hypotension, bradycardia, decreased respirations, and constricted pupils. This is the exact practice that drug abusers figured out when they started crushing sustained release oxycodone to get an opioid high.

There is no reason that he should be advised that he should consider going into a nursing home where they would be able to monitor his medications (choice A). It seems that he is doing fine at home, and that he just needs to be told that it is not safe for him to split the sustained release oxycodone and possibly, the other pills.

It is incorrect to tell him that he should have called you before he decided to begin the practice of “pill splitting” (choice B). Yes, maybe he should have called you and then you would have told him that this is not safe for all medications, but telling him that he “should have” called you seems accusatory and will make him defensive. It is best to just tell him that it is not safe. Putting the patient on the defensive is not the best way to maintain a good physician-patient relationship.

It is incorrect to tell him that this “pill splitting” practice is a good way to save money on his prescriptions (choice C). While it may be an acceptable, money saving practice for some pills, it is not adviseable for all pills. Some examples of the types of pills that should not be split are non-scored pills, enteric coated pills, and pills that are a sustained/controlled/extended release formulation.

You are going to stop prescribing the oxycodone if he continues to split the pills (choice E) is not the best answer because you should first try to explain to him that “pill splitting” is definitely not safe for the sustained release oxycodone pill that he is taking.

 

 

A 1-year-old child is brought to the clinic for a routine child health examination. His parents have been very compliant and have not missed any of his other health maintenance visits. He is a healthy child with no significant past medical history. Thus far, he has received the following vaccines (at the appropriate times): inactivated polio (IPV) 3 times, diphtheria/tetanus/acellular pertussis (DTaP) 3 times, hepatitis B (hep B) 3 times, haemophilus influenza type B (Hib) 3 times, and Pneumococcal conjugate (PCV) 3 times. At this time you should administer

  A. DTaP, Hib, MMR, varicella
  B. DTaP, IPV, varicella, PCV
  C. DTaP, MMR, varicella, PCV
  D. Hib, MMR, varicella, PCV
  E. IPV, Hib, MMR, PCV
Explanation:

The correct answer is D. Thus far this patient is up to date on his vaccines. The two live vaccines, MMR and the varicella vaccine, are not recommended for use before the first birthday, but should be given the first time the child sees the doctor after 12 months of age. The MMR then requires a booster dose between the ages of 4 and 6. If the varicella vaccine is given at less than 13 years of age, no booster is needed. PCV is a newer vaccine that is now recommended for all children between the ages of 2 and 23 months. If started at 2 months, the series consists of 3 primary vaccines, at 2, 4, and 6 months with a booster dose between 12 and 15 months of age. The primary series of Hib consists of 3 shots at 2, 4, and 6 months of age with a booster at 12-15 months. Although some formulations of the Hib vaccine require only 2 shots in the primary series (at 2 and 4 months of age), no matter what formulation is used, the booster dose must be given no earlier than 12 months of age, a minimum of 2 months after the previous dose.

DTaP, Hib, MMR, varicella (choice A) is not correct because this child is not due for DTaP at this time. The primary series for DTaP consists of 3 shots; 2 mo, 4 mo, and 6 mo. The first booster is given at 15 months of age and a minimum of 6 months must have elapsed between the third and fourth. Another booster is given at 4-6 years of age. After that time, the child should receive Td booster starting at 11-12 years of age and every 10 years thereafter.

DTaP, IPV, varicella, PCV (choice B) is not correct because he is not due for DTaP or IPV at this time.

DTaP, MMR, varicella, PCV (choice C) is incorrect because he is not due for DTaP at this time.

IPV, Hib, MMR, PCV (choice E) is incorrect because this child has already received 3 doses of IPV. IPV is given at 2 months, 4 months, and between 6 and 18 months. Another booster is not required until the child is between 4 and 6 years of age.

 

 

A 71-year-old man comes to the emergency department because of shortness of breath and a cough. He has been feeling short of breath for the past 5 days and has had a dry cough for 3 days. His past medical history includes diabetes, asthma, and hypertension. He has smoked two packs of cigarettes daily for the past 50 years. He denies alcohol use. Vital signs are: temperature 37.2 C (99.0 F), blood pressure 145/80 mm Hg, and pulse is 80/min. His oxygen saturation on room air is 90%. Auscultation of the lungs reveals diffuse wheezing. A chest x-ray is performed and demonstrates hyperinflated lungs, flattened diaphragms, and no evidence for infiltrates. The patient is placed on 30% supplemental oxygen and nebulized albuterol by face mask. Thirty minutes later, his oxygen saturation is 96% and he feels less short of breath. There are a few scattered wheezes on lung examination. Smoking cessation should be encouraged to this patient because

  A. it can prevent development of chronic obstructive lung disease (COPD)
  B. one year after quitting smoking, the risk of myocardial infarction is reduced by 10%
  C. the patient has end-stage lung disease
  D. the patient likely has lung cancer
  E. the risk of dying is reduced substantially even among persons who stop smoking after age 70
Explanation:

The correct answer is E. Smoking cessation should be encouraged to all patients who smoke, regardless of age and how long they have smoked. Smokers who quit smoking before the age of 50 have up to half the risk of dying in the next 15 years compared to people who continue to smoke. The risk of dying is reduced substantially even among people who stop smoking after age 70. After 10 years of abstinence, the risk of lung cancer is 30-50% lower than that of continuing smokers. Smokers who quit also have decreased chronic obstructive lung disease (COPD) related mortality rates, respiratory symptoms such as cough and wheezing, and infections such as bronchitis and pneumonia.

Smoking cessation to prevent the development of chronic obstructive lung disease (COPD) (choice A) is too late for this patient. His chest x-ray findings of hyperinflated lungs and flattened diaphragms are consistent with COPD. Smoking cessation in the setting of COPD will, however, reduce the mortality due to COPD as well as reduce respiratory symptoms and infections. Smoking cessation should be encouraged to all patients who smoke, regardless of age and how long they have smoked.

One year after quitting smoking, the risk of myocardial infarction is not reduced by 10% (choice B). The risk is actually reduced by 50%. Two years after quitting, the risk of stroke starts to decrease, and within 5-15 years, the risk of stroke approaches that of people who have never smoked. Smoking cessation should be encouraged to all patients who smoke, regardless of age and how long they have smoked.

This patient does not have end-stage lung disease (choice C). End-stage lung disease is a general term that describes advanced fibrosis of the lung parenchyma due to multiple etiologies including smoking and interstitial lung disease. Patients with end-stage lung disease will often require supplemental oxygen at all times and have CT findings of a “honeycombing” pattern throughout the lungs. This patient does have chest x-ray findings of chronic obstructive lung disease (COPD), but does not have end-stage lung disease. Patients with advanced lung fibrosis typically do not respond as quickly as this patient did to oxygen and albuterol.

There is no indication at this time that this patient has lung cancer (choice D). He is presenting to the emergency department with classic signs and symptoms of a chronic obstructive lung disease (COPD) exacerbation. The chest x-ray does not reveal any masses or lymphadenopathy to suggest a malignant process. Smoking cessation should be encouraged to all patients who smoke, regardless of age and how long they have smoked. Even with his long history of smoking, this patient can reduce his risk of lung cancer by quitting. After 10 years of abstinence, the risk of lung cancer is 30-50% lower than that of continuing smokers.

 

 

A 17-year-old boy is brought to the clinic by his father for a routine sports physical clearance. He has no current complaints. He is a junior in high school and plans to participate on the football, basketball, and track teams. He has no chronic medical conditions. He had an appendectomy at age 8. He takes no medications and develops a skin rash after taking penicillin. After his father leaves the room, you elicit more information regarding his health habits. He tells you that he lives at home and hopes to go to college on an athletic scholarship. He has smoked marijuana twice with his friends, but says that he does not like it. Almost every weekend he consumes between 4 and 10 beers at parties with his friends. He denies any feelings of anxiety or depression. He has a girlfriend and has had intercourse with her several times without using condoms. Physical examination is unremarkable. The intervention at this time that is most likely, if successful, to prevent the death of this patient within the next 5 years is

  A. counseling regarding the dangers of tobacco use
  B. discussion about avoidance of drinking and driving
  C. education about proper use of condoms to prevent sexually transmitted infections
  D. promoting a well-balanced diet high in anti-oxidants and monounsaturated fats
  E. recommending daily exercise and maintenance of a healthy weight
  F. screening for hypertrophic cardiomyopathy using an echocardiogram
Explanation:

The correct answer is B. Much of adolescent medicine centers on prevention. The leading cause of death for males aged 15-25 is motor vehicle accidents and approximately 50% of those accidents involve alcohol. It has been shown that preventive advice from physicians is effective in moderating the choices that teenagers make. Every adolescent visit should include a HEADDSS assessment, which stands for Home (interaction and strife within the family, who lives at home with the patient), Education (school performance, career goals), Activities (how the patient spends his/her free time, do they exercise, etc.), Drugs (including tobacco, alcohol, and illicit drugs), Diet, Sex (sexual activity, abuse, infections), and Suicide (depression, suicidal ideations or plans). Therefore, (choices A, C, D, and E) should be part of every visit. However, due to the excess mortality from motor vehicle accidents involving alcohol, this intervention is the most likely to prevent the death of this young man.

Screening for hypertrophic cardiomyopathy (choice F), is not correct. Although part of the sports physical is aimed at detecting patients who may die of sudden cardiac arrest with the goal of restricting them from participation in high-level sports, the number of patients affected by this disorder is small in comparison to those who die from accidental causes.

 

 

A 3-year-old boy is brought to the clinic by his parents who are concerned about the child’s behavior. He seems not to respond when they call him, even though he may be in another room playing. He does not show any emotion, if any of them hold him in their arms. Most of the time he stays in one place alone playing with a wind up train he has. He says a few words, but not all of them are clear and he does not seem to complete his sentences. He was born after a normal pregnancy without complications and his development so far has been unremarkable. He is well nourished and otherwise healthy, but the parents describe that he has not engaged in playing with other kids. In your office he establishes poor eye contact, but notices a box of crayons. He grabs them and keeps arranging them and throwing them around. He gets extremely upset when his mother tries to take the box away from him. At this time, the most appropriate next step in management is to

  A. order a brain CT
  B. refer him for a hearing exam
  C. refer him for speech therapy
  D. refer him to a child psychiatrist immediately
  E. order serum thyroid stimulating hormone
Explanation:

The correct answer is B. It would be important, before anything else is done, to rule out possible hearing impairment. It could certainly cause problems in the development of speech, and would compromise communication. Even if this is a case of autism, it would still be the first test before anything else is done. Mental retardation and hearing impairments are frequently seen in congenital disorders.

A brain CT (choice A) should be done to rule out CNS lesions. The standard of care implies that cheaper and less invasive procedures need to be done first if they can rule out most common causes of the disorder. Thus, a hearing exam should be done first.

A referral to a speech therapist (choice C) should be done after a probable diagnosis is established and further evaluation and speech rehabilitation needs to be done.

Referral to a child psychiatrist (choice D) should be done after basic metabolic screening and other exams are done to rule out medical causes. It is certainly not the first thing to do.

Thyroid stimulating hormone (choice E) levels should be checked in order to rule out a congenital deficiency. The fact that child has no signs of cretenism and that development has otherwise been normal, precludes the immediate need for these tests.

 

 

You are seeing a 30-month-old boy for well-child care. His parents are anxious about ensuring that his development is appropriate as he is their first and only child. He passed a hearing screen at birth and has been generally healthy besides a few colds. He has never been hospitalized or had any serious illness. He is able to run well, but has difficulty going down stairs. He uses more words than the parents are able to count and can use them in short, 2-word sentence fragments. It is difficult for you to understand a large part of what he is saying. He can draw a circle only if you show him how to do it. The most appropriate next step is to

  A. perform a brainstem auditory evoked potential hearing screen
  B. perform a screening exam for autism
  C. reassure the parents that his development appears normal and continue to follow up at well-child visits
  D. refer to a developmental specialist for comprehensive evaluation
  E. refer to speech-language therapy for evaluation of his language function
Explanation:

The correct answer is C. The developmental milestones mentioned in the vignette are within the range of normal for a 30-month-old boy. In the absence of any other evidence of significant impairment, there is no indication for referral at this point. One should always bear in mind though that developmental screening tests are neither sensitive nor specific, and if there are reasons for concern, such as marked impairment in one task or significant variations from the normal pattern of development (early handedness, attainment of milestones out of the normal order), a referral may be indicated despite passing a developmental screen.

A brainstem auditory evoked potential hearing test (choice A) may be indicated in infants who fail to meet language milestones if they cannot cooperate with a more comprehensive test. A 30-month-old should be able to cooperate with behavioral audiometry, so BAERs are not appropriate. In addition, this child has no evidence of language delay, and does not require referral at this point.

Autism (choice B) is an increasingly diagnosed cause of developmental delay, but this child is not delayed and no mention is given of any autistic traits, such as a lack of symbolic play, repetitive movements, or poor sociability.

Referral to developmental specialists or speech-language therapists (choices D and E) is not indicated for this normal child.

 

A 49-year-old man with paraplegia related to a gunshot wound in the military, but who is otherwise generally in good health, lives at the Veteran’s Administration nursing home. You see him for a yearly routine physical examination. He has no complaints. His family history is significant for a father who died at age 58 from an acute myocardial infarction, and a brother with a history of angina. He does not smoke and drinks alcohol socially. His blood pressure is 135/96 mm Hg, and the remainder of his physical examination is unremarkable. An electrocardiogram is normal. A fasting cholesterol profile is performed which shows:

The most appropriate management of this patient is

  A. a coronary arteriogram followed by coronary angioplasty
  B. metoprolol therapy and follow-up blood pressure in 3 months
  C. no therapy is required at this time as he has a normal cholesterol profile
  D. simvastatin therapy
  E. a trial of diet modification therapy, and then a follow-up lipid panel
Explanation:

The correct answer is E. This patient has multiple cardiac risk factors which include hyperlipidemia, mild hypertension, a positive family history, and male sex. The goal of the clinician is to modify the risk factors that can be altered. In this patient, these are his cholesterol and blood pressure. The initial therapy for both of these is dietary modifications and exercise. It often proves difficult to control blood pressure and cholesterol with these alone. However, it is always worth a try before subjecting the patient to life long drug therapy that can have multiple side effects. HMG CoA reductase (choice D) inhibitors and beta blockers (choice B) are some of the most effective and widely used cholesterol lowering agents and blood pressure lowering agents.

Although this patient may have coronary artery disease already, the patient is completely asymptomatic and an angioplasty (choice A) is not currently indicated.

It is not appropriate to not recommend any treatment whatsoever for this patient given his risk factors. And this certainly is not a normal cholesterol profile (choice C).

 

 

A 4-year-old girl is brought to the emergency department by her mother because of burns. The child was taking a bath and turned the hot water on when the mother stepped out of the bathroom to answer the telephone. When the mother returned, the child was sitting in extremely hot water up to her chest and crying. The child is now crying inconsolably. On physical examination, the skin over the child’s legs, lower abdomen, and pelvis are diffusely erythematous and swollen. There are a few small blisters on the dorsum of both feet. The skin on the chest and arms appear normal. The most appropriate step at this time is to

  A. consult child protective services for possible child abuse and neglect
  B. recommend an ice bath to reduce pain and swelling
  C. recommend quick showers instead of baths until the child is 10 years of age
  D. recommend setting hot water heater at 100 F or less
  E. recommend setting hot water heater at 130 F or less
Explanation:

The correct answer is E. The child has suffered first degree burns secondary to exposure to very hot water. Hot tap water burns account for 2,600 hospitalizations each year and 24% of all hospitalizations due to burns in children. These burn injuries are preventable by setting household water heaters at or below 120 to 130 F. Studies have shown that parents who received information on burn prevention during well-child care classes are more likely to have their hot water heaters set at 130 F or lower when checked by investigators during an unexpected home visit. This demonstrates the importance of providing this parent with counseling at this time to prevent future burn injuries in her household.

Consulting child protective services (choice A) is not necessary in this case. Hot tap water burns in children are unfortunately a very common injury. Many parents are unaware that these burn injuries can be prevented by simply setting the hot water heater at or below 120 to 130 F. Child abuse or neglect should be suspected in children who have a history of frequent injuries of different ages or who come to the emergency department multiple times with unusual injuries.

An ice bath to treat a first degree burn (choice B) is inappropriate for this patient as it is unnecessary and uncomfortable. The best care for a first degree burn is to run cool water over the burned area or hold a clean, cold compress on the burn until pain subsides. Aloe vera cream, aspirin, or ibuprofen may also help to alleviate pain. It is also critical to provide counseling for this parent to prevent future burn injuries in her household by setting the household water heater at or below 120 to 130 F.

Recommending showers instead of baths (choice C) is not the most appropriate counseling for this parent. It does not address the critical issue of hot tap water burn prevention. Setting household water heaters at or below 120 to 130 F has been shown to prevent this type of burn injury. Reducing the setting of the hot water heater thermostat will allow the child to safely take both showers and baths.

It is not necessary to reduce the hot water heater setting to 100 F or below (choice D).

 

 

A 14-year-old boy is brought to the office by his mother because of “bedwetting” episodes that have been occurring about twice a week for the past few months. The mother says that she noticed this “problem” when she washed his pajamas, and he “refused” to talk about it when she tried to bring it up. She is hoping that he will talk to you. You ask her to leave the room so you can have some privacy with her son. He starts the conversation by stating that “this is getting embarrassing” and he “doesn’t understand what’s going on.” He says that he gets up and finds his pajamas “wet and sticky.” He denies any dysuria or frequency during the day, and denies any problems at school or at home. He is on the basketball team, socializes with friends, and gets good grades. Physical examination is unremarkable and shows a pubic hair stage of Tanner IV and genital development Tanner stage III. The next best step is to

  A. advise him to stop drinking water at 8 pm and urinate before bed
  B. begin a 3-day treatment regimen with trimethoprim-sulfamethoxazole
  C. obtain a urine sample for culture and sensitivity
  D. order a renal ultrasound
  E. reassure him that this is a completely normal part of puberty
Explanation:

The correct answer is E. This adolescent boy is most likely having nocturnal emissions (“wet dreams”), which are a normal part of puberty. A nocturnal emission is when the penis becomes erect during sleep and ejaculates. If the boy or his mother do not know about nocturnal emissions before they occur, they may think that he urinated. He should be reassured that this is normal. It is also important to discuss it with his mother so she does not make him feel uncomfortable about it in the future.

It is not necessary to advise him to stop drinking water at 8 pm and urinate before bed (choice A) because he is having a nocturnal emission, which is not related to urination.

It is inappropriate to begin a 3-day treatment regimen with trimethoprim-sulfamethoxazole (choice B) because this boy is most likely having nocturnal emissions, not a urinary tract infection. Nocturnal emissions are normal and do not require treatment.

It is not necessary to obtain a urine sample for culture and sensitivity (choice C) or a renal ultrasound (choice D) because this boy is most likely having nocturnal emissions, which are not related to urinary tract infections (UTI). He does not have the symptoms of a UTI, which include frequency, dysuria, and nocturia. The nighttime “wetness and stickiness” that he describes is most likely due to semen, not urine.

 

 

A 20-year-old man with alpha-1 antitrypsin deficiency comes to the student health clinic for an initial visit when starting school. He tells you that he is here for a few years and would like to establish a relationship with the doctors in the clinic because of his chronic condition. He is currently asymptomatic. His vital signs and physical examination are normal. A chest x-ray reveals mildly increased lung volumes. Liver function tests are normal. Besides the routine counseling that you provide to all patients at the student health clinic, the most important preventive care issue for this specific patient is

  A. avoidance of alcohol and tobacco
  B. avoidance of accidental trauma
  C. keeping vaccinations current
  D. safe sex practices
  E. seatbelt use while driving
Explanation:

The correct answer is A. Patients with alpha-1 antitrypsin deficiency are prone to developing pulmonary emphysema and cirrhosis of the liver. Tobacco and alcohol use can greatly accelerate these processes. Given the prevalence of tobacco and alcohol in the college-age population, these are critical issues to discuss with this patient.

Trauma (choice B) is an important issue to discuss with all young patients, as it is the leading cause of death in this age group. It must be part of the routine counseling that you provide to all patients at the student health clinic. In this particular patient, however, the increased risk of alcohol and tobacco in the setting of alpha-1 antitrypsin deficiency must be discussed.

A majority of recommended vaccinations (choice C) have been administered at this time. Determination of vaccination history is a routine part of an exam. In this particular patient, as the question asks, the increased risk of alcohol and tobacco in the setting of alpha-1 antitrypsin deficiency must be discussed.

Safe sex practices (choice D) are particularly important in the college age patient. It must be part of the routine counseling that you provide to all patients at the student health clinic. In this particular patient, the increased risk of alcohol and tobacco in the setting of alpha-1 antitrypsin deficiency must be discussed, aside from the routine counseling.

Seatbelt use (choice E) is an important issue to discuss with all young patients, as motor vehicle accidents are the most common cause of death in this age group. This must be included in all counseling of patients in the student health clinic. In addition to routine counseling, due to his condition, the increased risk of alcohol and tobacco in the setting of alpha-1 antitrypsin deficiency must be discussed.

 

 

A 6-month-old infant is brought to the office for a routine well-child check. The father says that everything is going well at home, and that she is eating, sleeping and “progressing developmentally, just like the other kids.” She is up-to-date on her immunizations. Physical examination is unremarkable. The father tells you that they recently got a new family car with both driver and passenger side airbags. The infant seat that they have is a rear-facing infant seat. At this time you should

  A. advise him to disconnect the passenger-side airbag and place the infant in the infant seat on the passenger side of the front seat
  B. advise him to put the infant in the rear-facing infant seat on the passenger side of the front seat
  C. inform him that the safest seating position for the infant in the car is the middle of the rear seat
  D. recommend that the infant wear a helmet in the car to reduce the risk for a serious head injury
  E. tell him that airbags, in conjunction with lap and shoulder belts, have not been shown to reduce motor vehicle crash-related morbidity and mortality
Explanation:

The correct answer is C. Since motor vehicle injuries are a leading cause of death in children and young adults, the United States Preventive Services Task Force (USPSTF) recommends that counseling to prevent these injuries, is an important part of all periodic health maintenance examinations. Infant safety seats are important for all infants, and it is part of the physician’s responsibility to inform the parents as to the safest way to use these seats. It has been shown that the middle of the rear seat is the safest place in the car for the car seat to be placed. The American Academy of Pediatrics states that all infants and children under 12 years of age should ride in the back seat of the car and infants must always ride in rear-facing car seats in the back seat until they are at least 20 pounds and 1 year of age. It may seem silly to you that you need to counsel patients about things like this, but since accidents are one of the leading causes of death, counseling about the prevention of these accidents is one of the most important parts of your job.

It is incorrect for you to advise him to disconnect the passenger-side airbag and place the infant in the infant-seat on the passenger side of the front seat (choice A). While it is true that it is not recommended to place an infant in a rear-facing safety seat in the passenger side of the front seat with an airbag, you should tell him that the safest seating position in the car is the middle of the rear seat. If he tells you that for some reason he needs to always have the baby in the front seat in a rear-facing seat, then he should consider doing something about it. It is important for you to understand that airbags, in conjunction with lap and shoulder belts, have been shown to reduce motor vehicle crash-related morbidity and mortality. Since it is likely that the spouse and other family members will ride in the car, advising him to disconnect the airbag at this time is not the best choice.

It is inappropriate to advise him to put the infant in the rear-facing infant seat on the passenger side of the front seat (choice B). The USPSTF specifically says that rear-facing infant safety seats should not be placed in the front seat of a car equipped with a passenger-side air bag.

The USPSTF recommends that all children riding bicycles and people on motorcycles wear helmets. They do not recommend that infants wear helmets in the car to reduce the risk for a serious head injury (choice D). Infants should be in safety seats in the car.

It is incorrect to tell him that airbags, in conjunction with lap and shoulder belts, have not been shown to reduce motor vehicle crash-related morbidity and mortality (choice E). Airbags, used with lap/shoulder belts, have been shown to reduce morbidity and mortality.

 

 

A 33-year-old man is in the hospital recovering from a motor vehicle accident from which he sustained a liver laceration, right pneumothorax, and a splenic laceration. He is 2-weeks status post an exploratory laparotomy with splenectomy. His postoperative course has been unremarkable and he is now preparing to return home. He describes feeling “well”. His only current medication is a daily multivitamin. His temperature is 37.0 C (98.6 F), blood pressure is 124/78 mm Hg, pulse is 78/min, and respirations are 21/min. His cardiac rhythm is regular and his lungs are clear to auscultation bilaterally. Prior to his discharge, the patient should receive a vaccination against the

  A. Bordetella pertussis
  B. hepatitis B virus
  C. influenza virus
  D. Streptococcus pneumoniae
  E. varicella virus
Explanation:

The correct answer is D. The spleen constitutes a major site of immune response against encapsulated organisms such as Streptococcus pneumoniae, (pneumococcus). Patients following traumatic splenectomies are especially at risk for sepsis with such organisms. Therefore, this population should be routinely vaccinated against encapsulated organisms including pneumococcus.

Multi trauma victims are not at an elevated risk of developing pertussis. It is also likely that this patient was adequately immunized against pertussis with the DPT vaccine as a child. Therefore, the Bordetella pertussis vaccine (choice A) is not warranted at this juncture.

Multi trauma victims are not at an elevated risk of acquiring hepatitis B infection, (blood products are adequately screened in the U.S.). Therefore, the hepatitis B vaccine (choice B) is not routinely used in their care.

Young patients, even with a history of multi trauma, are not at an elevated risk of influenza. Therefore, the influenza vaccine (choice C), especially given the season (mid-summer), is not warranted in this situation.

Multi trauma victims are not at an elevated risk for developing varicella. Therefore, the varicella vaccine (choice E) is not warranted in this instance.

 

 

One winter evening you are evaluating a 68-year-old woman who is complaining of shortness of breath in the emergency department. She has a medical history significant for chronic obstructive pulmonary disease (COPD) and hypertension. Her medications include an ipratropium and albuterol inhaler and furosemide. Following a series of albuterol nebulizer treatments, her respiratory function returns to baseline. Her temperature is 37.0 C (98.6 F), blood pressure is 146/87 mm Hg, pulse is 89/min, and respirations are 22/min. She has diminished breath sounds bilaterally, but otherwise her lungs are clear to auscultation. Prior to her discharge, she should receive

  A. a tetanus booster
  B. vaccination against hepatitis B
  C. vaccination against influenza
  D. vaccination against Streptococcus pneumoniae
  E. vaccination against varicella
Explanation:

The correct answer is C. Elderly patients with underlying lung disease are at high risk for severe influenza infections during the wintertime. Therefore, this patient population must receive their influenza vaccine in order to reduce their risk.

A tetanus booster (choice A) is not warranted since this patient has no history of trauma and exposure to contaminated elements (soil etc.)

Vaccination against hepatitis B (choice B) is not warranted in this instance since the patient has no history of exposure to possible contaminated products such a blood or bodily fluids.

Advanced age and underlying lung disease are not specifically associated with an elevated risk of infection with Streptococcus pneumoniae. Therefore, vaccination against Streptococcus pneumoniae(choice D) is not warranted in this instance.

Vaccination against varicella (choice E) is not warranted since there is no history of exposure to the varicella zoster virus during the current admission or recent past.

 

 

A healthy 12-year-old boy is brought to the clinic for a sports participation physical examination. He is planning on trying out for the junior varsity football team. He is going into seventh grade and his school recently switched the high school to seventh through twelfth grade. His mother tells you that he is very concerned because he has “not yet reached puberty” and he is afraid that he is going to be “harassed” by the older boys. Physical examination shows no pubic hair and a preadolescent penis and testes. The remainder of the physical examination is unremarkable. The mother and patient want to know “what is going on.” At this time you should

  A. advise him that he should consider taking male hormones, such as testosterone
  B. explain that puberty usually starts by 10 years old and that he is most likely developmentally delayed
  C. recommend that he go for genetic testing to evaluate him for a genetic disease
  D. tell him that he should be looking for testicular enlargement as the first sign of puberty
  E. tell them that everybody is different and there is no particular, predictable pattern of development
Explanation:

The correct answer is D. Puberty usually refers to the time between the development of secondary sexual characteristics and rapid growth and the end of somatic growth. It may occur at different times, but often follows a predictable pattern. The first sign of puberty in boys is testicular enlargement, which often occurs at 11.6 years of age. 11-12 years old is the average age of pubertal development in boys. But the normal range is 9-14 years. Therefore, this boy needs reassurance that he is within the normal range and since he is anxiously awaiting puberty, he should be told what to look for as the first sign.

Advising him that he should consider taking male hormones, such as testosterone (choice A) is inappropriate because he is well within the normal range of development at this time. The average age of the onset of puberty is 11-12 years, but the range is from 9-14 years.

It is incorrect to explain that puberty usually starts by 10 years old and that he is most likely developmentally delayed (choice B) because the average age of onset is 11-12 years, but the range is from 9-14 years. It is premature to tell him that he is delayed.

Recommending that he go for genetic testing to evaluate him for a genetic disease (choice C) is incorrect because he is only 12 years old and the normal range for the onset of puberty is 11-12, and the range is 9-14 years. If the remainder of the physical examination is unremarkable, and he has had no previous problems, genetic testing at this time is inappropriate.

Even though development may occur at different ages, there is often a predictable pattern of pubertal development called Tanner stages. It is therefore incorrect to tell them that everybody is different and there is no particular, predictable pattern of development (choice E). In males, stage I refers to a preadolescent with no pubic hair, stage II refers to testicular enlargement and scant pubic hair, stage III refers to penile enlargement, greater testicular enlargement and curling of pubic hair, stage IV refers to greater penile enlargement, darkening of the scrotum, and adult-type pubic hair (but less of it), and stage V refers to adult size penis and testes and adult distribution of pubic hair.

 

 

A 22-year-old college student comes to the student health clinic for an annual physical examination and to renew her prescription for birth control pills. She has no medical problems and her only medications are oral contraceptive pills and occasional ibuprofen for headaches. She feels well and reports infrequent alcohol consumption and denies cigarette smoking. She has a family history of breast cancer in her mother who was diagnosed at age 45 and recently died of metastatic breast cancer. The patient’s vitals and physical examination, including breast examination, are normal. The most important preventive care measure to recommend for this patient is

  A. annual clinical breast examination (CBE)
  B. annual mammogram starting immediately
  C. monthly self breast examination (SBE)
  D. monthly self breast examination (SBE) and annual mammogram starting at age 35
  E. monthly self breast examination (SBE) and annual mammogram starting at age 40
Explanation:

The correct answer is D. All women should be encouraged to practice monthly self breast examinations (SBE) in order to become familiar with their breasts and to be aware of any changes. The American Cancer Society, the American College of Radiology, the American Medical Association, the American College of Obstetricians and Gynecologists, and a number of other organizations recommend screening with mammography and annual clinical breast examinations (CBE) beginning at the age of 40. The recommendations are different for women who have a first degree relative with breast cancer. The American College of Radiology recommends an annual screening mammogram starting 10 years before the age at diagnosis of the youngest affected relative. The most important recommendation for this patient is both the SBE and an annual mammogram starting at age 35, which is 10 years before her mother was diagnosed.

An annual clinical breast examination (CBE) (choice A) is not the most important recommendation for this patient. The studies are unclear as to effectiveness of CBE in the detection of early breast cancer, particularly in younger women. The recommendations for women ages 20-39 is for a CBE every 1-3 years and monthly self breast examinations. The recommendations are different for women who have a first degree relative with breast cancer.

An annual mammogram starting immediately (choice B) is not appropriate for this patient. The American Cancer Society, the American College of Radiology, the American Medical Association, the American College of Obstetricians and Gynecologists, and a number of other organizations recommend screening with mammography and annual clinical breast examination (CBE) beginning at the age of 40. The recommendations are different for women who have a first degree relative with breast cancer. The most important recommendation for this patient is both the SBE and an annual mammogram starting at age 35, which is 10 years before her mother was diagnosed.

A monthly self breast examination (SBE) (choice C) is an important recommendation for this patient. However, since she has a first degree relative diagnosed with breast cancer at age 45, it is also recommended that she start having annual mammograms at age 35. The monthly self breast examination (SBE) is an important recommendation for this patient. Annual mammography and clinical breast examination (CBE) starting at age 40 (choice E) is recommended for women without a family history of breast cancer. Since this patient has a first degree relative diagnosed with breast cancer at age 45, it is recommended that she start having annual mammograms at age 35.

 

 

A 7-month-old infant is brought to the office for a well-child visit. The mother has been breastfeeding and says that everything is going well. He is on a regular feeding schedule and he is not eating any solid foods or receiving any supplements. He is up-to-date on all of his immunizations. The mother is proud of his development; he is able to sit alone without support, can smile at himself in the mirror, has a palmar grasp, and can make sounds with multiple syllables. His physical examination is unremarkable. At this time you should

  A. advise the mother to begin giving him vitamin C supplements to prevent scurvy
  B. advise the mother to start giving him iron supplements or iron-containing foods to prevent iron-deficiency anemia
  C. explain to the mother that exclusive breastfeeding is ideal nutrition and sufficient to support optimal growth and development until 12 months
  D. explain to the mother that it may be best to discontinue breastfeeding and give him a commercially prepared formula that contains all of the vitamins and minerals that he needs at this time
  E. schedule a follow-up visit for his 12-month visit when he will receive his first varicella and measles mumps rubella vaccinations
Explanation:

The correct answer is B. By 6 months of age, breast-fed infants should be given iron supplements or iron-containing foods to prevent iron-deficiency anemia. Until this time, breast milk typically provides an adequate amount of iron.

Vitamin C supplements (choice A) are not necessary to prevent scurvy in breast-fed infants.

Exclusive breastfeeding is ideal nutrition and sufficient to support optimal growth and development until approximately 6 months, not 12 months (choice C). After 6 months, iron-enriched foods or supplements should be added to prevent iron-deficiency anemia.

The American Academy of Pediatrics recommends that physicians encourage their patients to breastfeed for at least 12 months. Therefore, it is incorrect to tell her that it may be best to discontinue breastfeeding and give him a commercially prepared formula that contains all of the vitamins and minerals that he needs at this time (choice D). Supplementation with enriched foods is all that is necessary to provide optimal nutrition at this time.

While it is correct to schedule a follow-up visit for his 12-month visit when he will receive his first varicella and measles mumps rubella vaccinations (choice E), it is necessary that you first tell her that she needs to supplement the breastfeeding with iron to prevent iron-deficiency anemia.

 

 

A 42-year-old man comes to the emergency department with a history of a productive cough and hemoptysis. He is known to be HIV positive and admits to a history of intravenous drug abuse. In the emergency department, his temperature is 38.1 (100.6 F) with stable vital signs. He is admitted to the medical floor for the treatment of pneumonia. A chest x-ray and subsequent CT scan of the chest confirms a cavitary lesion in the right lung. He is started on antibiotics and sputum is sent for a Gram stain, acid-fast bacillus smear, cultures, and sensitivity. The acid-fast bacillus smear comes back positive. He now admits that he was diagnosed with pulmonary tuberculosis 4 years ago and was advised treatment with isoniazid (INH), rifampicin, pyrazinamide, and ethambutol, which he was supposed to take for 6 months. His compliancy in taking these medications is unclear. From the period of his admission to the emergency department and placement on respiratory isolation, several hospital employees were exposed to his respiratory secretions. A PPD test is given to all the exposed employees. Three employees with previously negative PPD test results, now have positive results.The most appropriate post exposure prophylaxis (PEP) plan for these employees is

  A. ethambutol and pyrazinamide therapy initially, then the addition of this regimen accordingly to the patient’s sensitivity profile
  B. to hold postexposure prophylaxis until the patient’s sensitivity profile is available and then choose a regimen
  C. INH for 6 months,and if PPD is still positive, an extension of the treatment for 12 months
  D. INH, rifampicin, pyrazinamide, and ethambutol for 6 months and recheck the PPD
  E. no postexposure prophylaxis is necessary
Explanation:

The correct answer is B. Tuberculosis exposure is not an emergency and does not require immediate re-initiation of postexposure prophylaxis. Given the potential toxic side effects of antituberculosis medications and the importance of completing a regimen for full efficacy, it is reasonable to withhold initiation of postexposure prophylaxis for tuberculosis until sensitivities are available. Tuberculosis bacilli sensitivities usually take a few weeks to come back. This particular patient is at high risk for multidrug resistant tuberculosis, because he was non-compliant with different medications for tuberculosis in the past. In the case of multidrug resistant tuberculosis, postexposure prophylaxis should consist of 2 drugs to which the index patient’s tuberculous strain is susceptible.

Ethambutol and pyrazinamide by themselves may not completely cure tuberculosis, unless the patient’s strains are sensitive to this antituberculosis medication (choice A).

INH by itself has shown to be effective in treating tuberculosis. But INH resistant strains are well known, and these patients should be treated with additional rifampicin (choice C).

A combination of 4 different medications (choice D) may be essential depending upon the tuberculosis sensitivity profile, but one should wait before starting this therapy to avoid any side effects and development of drug resistant tuberculosis.

Postexposure prophylaxis is essential after the sensitivity profile is known (choice E).

 

 

A 9-year-old girl is brought to the emergency department by her parents after she fell off her bicycle while riding in the street. She is crying and has an obvious deformity of her left wrist. On physical examination, she also has scalp swelling over the left frontal skull and a 3 cm laceration on the left side of her forehead. X-rays of the left wrist demonstrate a non-displaced fracture of the distal radius. A CT of the head reveals soft tissue swelling over the left frontal skull and a non-depressed fracture of the underlying calvarium. The most important next step is to

  A. contact child protective services regarding potential child abuse
  B. discourage bicycle riding and encourage other sports
  C. encourage safety helmet use
  D. encourage safety helmet use and bicycle safety training
  E. recommend that the child ride her bicycle only when accompanied by an adult
Explanation:

The correct answer is D. Bicycle related injuries are a large and important cause of morbidity and mortality in school age children. It is very important to recommend the use of safety helmets while bicycling. Head trauma, such as the skull fracture in this case, accounts for approximately 50-85% of bicycle fatalities and hospitalizations. Multiple studies of bicycling accidents suggest that bicycle helmets reduce head injuries by 60-80%. Bicycle safety training is another useful intervention to promote bicycle safety and is suggested by surveys that demonstrate that many bicycle accidents among children result from cyclist error. One small controlled trial in children aged 8 to 9 years reported a positive short-term effect of safety training on bicycling behavior.

Contacting child protective services regarding potential child abuse (choice A) is not appropriate at this time. There is no clinical indication that the child has been abused. Many school age children suffer bicycle related injuries, particularly if they neglect to wear a safety helmet. It is very important to recommend a safety helmet as well as to encourage bicycle safety training. Child abuse should be suspected if a child suffers from multiple injuries of differing ages or if there are numerous visits to the emergency department with unusual injuries.

Discouraging bicycle riding (choice B) is unnecessary. Although bicycle related injuries are an important cause of morbidity and mortality in children, the risk of injury can be greatly reduced with appropriate safety gear and training.

Encouraging safety helmet use (choice C) is an important part of bicycle safety in school age children. It is also important to recommend bicycle safety training in addition to safety helmets.

Recommending that the child ride her bicycle only when accompanied by an adult (choice E) is unnecessary and inappropriate. The most important recommendation to make in light of a recent bicycle injury with associated head trauma is the use of safety helmets. It is also important to encourage bicycle safety training in addition to safety helmets.

 

 

A young mother brings her 18-month-old son into your office for a well-child examination. She is concerned that he “seems to be slow for his age.” His birth was without incident and his history is remarkable only for an episode of otitis media at 7 months that responded well to antibiotics. He has a 5-year-old sister at home and lives with both parents and a pet dog. When asked to describe her concern more fully, the mother simply states, “He just isn’t doing the same things that his sister did when she was this age.” She reports that he is walking without help, gives hugs and kisses to family members, is able to feed himself with a spoon, and has a vocabulary of about 15 words. However, he is unable to climb stairs by himself, cannot turn a doorknob to open a door, and shows no interest in potty training. His height is 65th percentile and weight is 75th percentile. He does not speak to you, but answers simple questions by nodding or shaking his head. You elicit no specific findings on physical exam. The most appropriate response to the mother’s concerns is:

  A. “His lack of social involvement may indicate autism. I’d like to send you to a pediatric psychiatrist for further evaluation.”
  B. “I think that your son should be referred to a developmental specialist since I’m concerned that he shows some delay in motor skills.”
  C. “I’d like to perform some audiology tests since your son should be speaking in short phrases by now.”
  D. “There is no need to be concerned; your son is right where he should be as far as his development.”
  E. “You should begin toilet training immediately, he’ll find it extremely difficult if you don’t start before age 2.”
Explanation:

The correct answer is D. By the age of 18 months, most children will have developed a specific set of skills which can be divided into: social, self-help, gross motor, fine motor, and language. Social skills at this age include greeting people by saying “hi” or something similar, giving hugs and kisses, and playing patty-cake. Self-help involves drinking from a cup and feeding self with a spoon. Gross motor skills include walking without help and beginning to run. Fine motor skills would include scribbling with a crayon and stacking 2 blocks. Language skills should encompass talking in single words, asking for food or drink with words, and following simple instructions. This patient is doing well regarding each of these milestones.

“His lack of social involvement may indicate autism. I’d like to send you to a pediatric psychiatrist for further evaluation” (choice A) is incorrect since the patient does not exhibit any signs of autism.

Referral to a developmental specialist (choice B) is inappropriate since the patient’s motor skills are at a level with his age. Most children cannot walk up and down stairs alone or open a door using a doorknob until 2 years of age.

An audiology referral (choice C) is incorrect. By 20 months of age a child should know at least 10 words. This patient is able to use 15. Most children do not speak in phrases until after age 2.

“You should begin toilet training immediately, he’ll find it extremely difficult if you don’t start before age 2,” (choice E) is incorrect since the majority of children are not toilet trained until 3 years.

 

  A 64-year-old African American man comes to the clinic for a periodic health maintenance examination. He is currently asymptomatic, has no significant past medical history, and takes no medications. He is married with 2 children, works as an accountant, and does not smoke. He drinks 5 alcoholic drinks per week. This patient is most at risk for

  A. accidental trauma
  B. coronary artery disease
  C. head-and-neck cancer
  D. Human Immunodeficiency Virus (HIV)
  E. osteogenic sarcoma
  F. suicide
Explanation:

The correct answer is B. Coronary artery disease is the most prevalent disease in this age group. Cancer is also prevalent in this age group with lung, prostate, and colon cancer being the most common. Keep in mind the difference between prevalence and incidence when answering this sort of question. Prevalence is the percent of a given population effected at any given time. Incidence is the number of new cases arising for a period of time.

Accidental trauma (choice A) is the leading cause of death in the under 30 age group.

HIV (choice D) is prevalent in younger populations particularly in groups that have multiple sexual partners and use intravenous drugs.

Head-and-neck cancer (choice C) is one of the most common causes of cancer-related death, but would be less common than coronary artery disease in this age group.

Osteogenic sarcoma (choice E) occurs in the second to fifth decades of life in long tubular bones and would be relatively rare in this population.

Suicide (choice F) is second only to accidental trauma as the cause of death in the under 30 age group.

 

A 71-year-old man comes to the office for a periodic health maintenance examination. He is a long-standing patient and was last seen by you almost 14 months ago. He has diet-controlled diabetes mellitus, hypertension, glaucoma, mild peripheral vascular disease, and osteoarthritis. His medications include lisinopril, atenolol, aspirin, and acetaminophen as needed for pain. He has no complaints and his review of systems is negative for any chest pain, orthopnea, paroxysmal nocturnal dyspnea, headache, visual changes, calf pain, or abdominal pain. He has no allergies and he smokes two packs of cigarettes per day. His temperature is 37.0 C (98.6 F), blood pressure is 160/80 mm Hg, pulse is 61/min, and respirations are 16/min. Physical examination is notable for a left carotid bruit, a 2/6 systolic ejection murmur heard best at the left sternal border, and clear lungs. His abdomen is soft with no masses, but there is a previously appreciated abdominal bruit. He received his pneumococcal vaccine three years ago and his MMR and Td boosters at age 51. The most appropriate immunization at this time is

  A. hepatitis B vaccine
  B. influenza vaccine
  C. pneumococcal vaccine
  D. tetanus diphtheria vaccine
  E. varicella vaccine
Explanation:

The correct answer is B. There is reasonably good evidence that all senior citizens (age 65 or greater) should receive the influenza vaccine regardless of their risk status. Lower risk patients have a smaller absolute risk reduction in hospitalization due to pulmonary problems and mortality but there is still a sizable medical benefit.

There is no indication for hepatitis B vaccine (choice A) unless the patient is a health care worker or HIV positive.

The pneumococcal vaccine (choice C) is generally given just once every five years after the age of 50 if the patient has a chronic disease. This patient last received his three years ago.

The recommendations for a Td vaccine (choice D) are every 10 years or once at age 50.

Unless the patient is also involved in day care or is a teacher, there is no recommendation for varicella vaccine (choice E).

 

 

You are notified that one of your patients, a 76-year-old woman with hypertension, osteoporosis, and hypothyroidism, is being discharged from the hospital where she has been treated for an acute subdural hematoma sustained after a fall 2 weeks ago. A quick review of her hospital chart reveals that she had an uneventful stay with her blood pressures ranging from 125/90 mm Hg to 140/90 mm Hg, and pulse from 60/min to 70/min. You review her current medications and note that she is taking hydrochlorothiazide, alendronate sodium tablets, and thyroxine. You schedule a visit with the patient and her son to establish a discharge plan. She tells you that she lives alone and is planning on going back to her own apartment, not to “nursing home hell.” Her son says that he will make sure that she has some type of part-time home care. The most appropriate intervention to prevent another fall is to

  A. discontinue her medications
  B. keep phones at floor level
  C. recommend a daytime home health care worker
  D. schedule a home safety evaluation
  E. tell her to walk around in stockings or socks at home
Explanation:

The correct answer is D. Falls are one of the most important problems in the elderly population. Approximately one third of individuals over the age of 65 fall annually. Falls are the most important cause of injury in this age group. There are many factors that usually lead to a fall, both intrinsic and extrinsic, and include medications, a medical illness, foot problems, and a “hazardous” home environment. After an acute medical illness is ruled out or treated, targeted interventions must occur. Targeted interventions include a full medication review with the elimination of unnecessary drugs, physical therapy, and a home safety evaluation. Warped floorboards, inadequate lighting, and throw rugs may all contribute to an unsafe home environment.

Discontinuing her medications (choice A) is inappropriate as she has a specific need for each of these agents. It is possible to switch some of her medications, like possibly try a different antihypertensive agent. However, the thyroxine for hypothyroidism and the alendronate for osteoporosis, should not be discontinued.

Keeping phones at floor level (choice B) is recommended so that the patient can have easy access if a fall occurs. This way they can get help as soon as possible. However, this does not prevent a fall.

A daytime home health care worker (choice C) may be useful when they are there, but she will be at greatest risk for a fall at night. Also, the worker cannot keep an eye on her every second. Therefore, a home safety assessment is a better answer.

Telling her to always walk around in stockings or socks at home (choice E) is inappropriate because they are slippery and can increase the risk of a fall. Your goal is to prevent another fall, not to cause one.

 

 

A 67-year-old retired pharmacist is brought to the emergency department by ambulance after being involved in a motor vehicle accident. He rear-ended another car while traveling at approximately 30 mph. He was not wearing a seat belt at the time. At the scene of the accident, the patient reported feeling mild neck pain. On physical examination, he has a bruise on his forehead, but no lacerations. X-rays of the cervical spine are normal. A CT of the head is also normal. The patient reports occasional alcohol use and denies any drug use. He says that he had 1 glass of wine with dinner about 3 hours before the accident. The most important recommendation for this patient is to

  A. buy a car with driver side airbags
  B. buy a car with both driver side and passenger side airbags
  C. wear a seat belt only when driving, as it is not mandatory as a passenger
  D. wear a seat belt while driving even if the car has airbags
  E. wear a seat belt while driving only if the car does not have airbags
Explanation:

The correct answer is D. The most important recommendation for this patient is to wear a seat belt at all times in a car, whether he is a driver or passenger. This is true whether the car has airbags or not. Use of safety belt restraints has been shown to reduce the risk of motor vehicle injury and death. Studies have shown that lap and shoulder belts can decrease the risk of moderate to serious injury to front seat occupants by up to 55% and can reduce crash mortality by 40-50%. Motor vehicle accident victims who are wearing seat belts at the time of the crash are less likely to require hospital admission and have lower hospital charges. Multiple studies that evaluated mandatory seat belt laws report significant reductions in motor vehicle related injuries, hospital admissions, and fatalities after implementation of such laws.

Buying a car with driver’s side airbags (choice A) is not the most important recommendation for this patient. He was not wearing a seat belt at the time of his accident and this is a good opportunity to encourage him to do so at all times while in a car. Although it is not mandatory to drive a car with airbags, it is mandatory to wear safety belts in 49 states. Forty-nine states (all except New Hampshire) and the District of Columbia have mandatory safety belt laws. In most states, these laws cover front-seat occupants only. Beginning with the model year 1998, all new passenger cars in the U.S. are required to have driver and passenger side air bags. A review by the National Highway Traffic Safety Administration estimated that air bags increase the effectiveness of safety belts by about 5-10%.

Buying a car with both driver side and passenger side airbags (choice B) is not the most important recommendation for this patient.

Wearing a seat belt only while driving and not as a passenger (choice C) is inappropriate counseling. It is important to advise the patient to wear a seat belt at all times while in a car as a driver or passenger.

It is inappropriate to advise the patient to only wear a seat belt if the car does not have airbags (choice E). Airbags are designed as a safety supplement to seat belts and are not meant to be used as a substitute. .

 

 

A 56-year-old woman, who you have not seen in 6 years, comes to the office for a routine physical examination. She has no complaints. Her past history is remarkable for an ovarian cyst removal at age 19. Her last menstrual period was 4 years ago. Her father died of a myocardial infarction at age 59 and mother is alive and well. Her sister and her aunt both died of breast cancer at age 61. Her father and her grandfather both had diabetes. She appears her stated age, is well and in no distress. Her weight is 67kg (147 lb) and her height is 160 cm(5 ft 3 in.). Her temperature is 36.7 C (98.0 F), blood pressure is 110/70 mm Hg, and pulse is 60/min. Physical examination is unremarkable. Concerning her risk for ovarian cancer, the most appropriate next step in management is to

  A. determine CA-125 levels, annually
  B. determine CA-125 levels and perform a pelvic ultrasound, annually
  C. do nothing until she has symptoms consistent with ovarian cancer
  D. do pelvic examinations, annually
  E. perform pelvic ultrasounds, annually or biannually
Explanation:

The correct answer is C. Ovarian cancer continues to be a leading cause of death for women and most ovarian cancer is diagnosed in the later stages, making treatment or cure exceedingly difficult. Despite this fact and the widespread awareness of this problem by the public and physicians, there is currently no routine screening that is recommended for ovarian cancer.

Annual CA-125 testing (choice A) is incorrect. There is little data to date from a variety of trials that CA-125 screening would improve outcomes. It may seem reasonable to assume that combination testing may increase the utility of these tests.

There is little data that the combination of annual CA-125 combined with pelvic ultrasound (choice B) is superior to either test when used alone.

It has been estimated, being generous, that annual pelvic examinations(choice D) would reduce 5-year-mortality by less than 0.01%. Therefore, they are not routinely recommended for ovarian cancer screening.

Annual or biannual pelvic ultrasound (choice E), although seemingly a good idea, has a very, very poor specificity and has been estimated to be able to detect 4 cases of cancer in 10,000 women at a cost of 540 false-positive results.

 

 

A 47-year-old white man comes to the clinic for a routine periodic physical examination. He is a healthy man overall without any medical problems, and he does not have any complaints at this time. You notice that he appears more jittery than usual. Upon further questioning you realize that he is here because his 72-year-old father was just diagnosed with prostate cancer, and he is worried that he might be at risk as well. There have not been any other members of his immediate family who have been diagnosed with prostate cancer. His physical examination is normal and his routine laboratory studies, including cholesterol levels, are also unremarkable. The patient does not have any urinary symptoms and on digital rectal exam (DRE), his prostate seems smooth without any nodularity. You discuss with him that routine screening for prostate cancer is a controversial issue, but he adamantly insists that he “needs to know what is going on inside his body”. He explains that he has been doing some independent research on the Internet and realizes that different groups have different ideas about screening, but he is willing to pay for “whatever test can be done” to put his mind at ease. He has not slept in days because of his fear of cancer, and he has been performing poorly at work. He wants to know if he has the disease, so he can begin to treat it when he is relatively young and healthy, and “hopefully” before it metastasizes. After you explain the risks and benefits of the specific screening tests, the most appropriate next step is to

  A. check his prostate specific antigen level in 3 weeks since a digital rectal exam was just performed, and will lead to a false elevation
  B. check his prostate specific antigen level today
  C. reassure him that since he is at low risk because of his young age and normal digital rectal exam you cannot do any screening tests at this time
  D. refer him to a urologist for a possible prostate biopsy
  E. tell him that you will start yearly prostate specific antigen checks and digital rectal exams when he turns 50
Explanation:

The correct answer is B. The patient has a first-degree male relative who has been diagnosed with prostate cancer, which puts him at a slightly increased risk of developing prostate cancer. The American Urological Association recommends screening with an annual prostate specific antigen (PSA) check and DRE starting at age 40 for these men. It also recommends screening starting at age 40 for African American men, and at age 50 for everyone else. The United States Preventive Service Task Force says that routine screening of asymptomatic men is not recommended. However, each patient must be treated individually and each decision that the physician makes should be well thought out. This patient is becoming dysfunctional in his personal life and since his concerns are reasonable it seems best in this case to order a test, as long as he understands the risks and benefits. Therefore the next best step for this patient is to get a PSA.

Check his PSA level in 3 weeks since a DRE was just performed and will lead to a false elevation (choice A) is incorrect because a routine digital rectal exam does not lead to an elevation in PSA.

Reassure him that since he is at low risk because of his young age and normal DRE that you cannot do any screening tests at this time (choice C) is incorrect because the patient does have a first-degree relative diagnosed with prostate cancer and is at increased risk. Also, telling him that you “cannot do any screening tests at this time” is incorrect. You can order reasonable tests in specific cases where the patient is extremely worried about a realistic problem as long as he understands the risks and benefits.

Referral to the urologist for a biopsy (choice D) is incorrect because the referral to a urologist is premature and inappropriate at this time, especially without all the necessary information, including PSA levels.

Tell him that you will start yearly PSA checks and DREs when he turns 50 (choice E) is not the best choice because this patient seems very affected by the thoughts of cancer and is becoming dysfunctional. He is at an increased risk because of his father’s diagnosis and sometimes it is best to order tests that may not always be recommended for all groups, as long as the patient understands the risks and benefits.

 

 

A 15-year-old girl is brought to the office for a periodic health maintenance examination. She recently moved to your city and so this is the first time that you are meeting her and her mother. After you obtain a detailed medical history and her past immunization schedule from the patient and her mother, you tell the mother that you routinely talk and examine adolescents alone, without their parents in the room. The mother agrees to leave and the patient is willing to freely discuss her sexual history and her recreational drug use. You decide that STD screening and a Pap smear are important during this visit since she has had sexual intercourse with 3 different partners. After you complete the physical examination and before you begin the pelvic examination, you should

  A. advise her to relax, that it will not hurt because she is already sexually active
  B. ask if she would like her mother or another member of the health care team present as an observer
  C. explain that this is only necessary because she has had so many sexual partners
  D. put a lubricant on the speculum, insert the speculum, and obtain a Pap smear of the cervix
  E. tell her that she should consider HIV testing because she has been promiscuous
Explanation:

The correct answer is B. It is important to recognize that a pelvic examination may make this adolescent uncomfortable, even though she has been sexually active, and it is important to ask if she would be more comfortable if there is someone else present in the room. Therefore, you should ask if she would like her mother or another member of the health care team present as an observer. An adolescent girl may feel that another person in the room, in addition to the physician, is helpful in putting her mind at ease. Never assume that she wants her mother present for this. Always ask her alone if she wants her mother to be called back in. Many times the answer is no.

It is inappropriate to advise her to relax and that it will not hurt because she is already sexually active (choice A). You can tell her to relax, but definitely do not say that it will not hurt because she is already sexually active. You should not say this because it still may hurt, even if she is sexually active, and also, it sounds judgmental.

Do not explain that this is only necessary because she has had so many sexual partners (choice C). This is judgmental and untrue. A Pap smear is recommended for all sexually active adolescents, whether she has been with 1 partner or 10 partners.

It is incorrect to put a lubricant on the speculum, insert the speculum, and obtain a Pap smear of the cervix (choice D) because a lubricant may interfere with the Pap smear specimen.

You should not tell her that she should consider HIV testing because she has been promiscuous (choice E) because telling her that she is “promiscuous” sounds too judgmental. The United States Preventive Services Task Force recommends that the physician should assess risk factors for HIV infection in all patients by obtaining a careful sexual history and inquiring about drug use. HIV counseling and testing should be offered to all patients that are at increased risk for infection. This should always be done in a nonjudgmental way.

 

 

A 35-year-old graduate student is in the hospital following a major motor vehicle accident. He was an unrestrained passenger in a car that was sideswiped by another vehicle traveling at 50 mph. The driver of the car that the patient was in was drunk and instantly killed. The patient suffered multiple fractures and blunt injuries to the abdomen. He is postoperative day two following a splenectomy and an open reduction internal fixation of a left femur fracture. He has had a normal postoperative course and is feeling well. Prior to discharging him to a rehabilitation facility, it is most important to

  A. encourage the patient to wear a seat belt when he is the driver
  B. encourage the patient to wear a seat belt when he is the driver or passenger
  C. question the patient regarding whether he was drinking alcohol prior to the accident
  D. recommend buying a car with airbags
  E. recommend buying a car with airbags and encourage him to wear a seat belt when he is the driver
Explanation:

The correct answer is B. This patient was an unrestrained passenger in a car with a drunk driver. It is most important to counsel the patient about wearing a seat belt in a car at all times. The use of occupant restraints has been shown to reduce the risk of motor vehicle injury and death. Multiple studies have demonstrated that the proper use of lap and shoulder belts can decrease the risk of moderate to serious injury to front seat occupants by up to 55% and can reduce crash mortality by 40-50%. Crash victims who were wearing seat belts at the time of the accident have less severe injuries and are less likely to require hospital admission. In addition, it is also important to discuss the dangers of being in a car with a drunk driver.

It is not sufficient to counsel the patient to wear a seat belt when he is the driver (choice A). The patient should be encouraged to wear a seat belt in a car at all times.

Questioning the patient as to whether he was drinking alcohol prior to the accident (choice C) is not the most important discussion to have before discharge. The patient was an unrestrained passenger in car with a drunk driver. He should be counseled about wearing a seat belt in a car whether he is a driver or passenger. In addition, he should also be counseled about the dangers of being in a car with a drunk driver. Driving while impaired by alcohol or drugs and failing to use occupant protection such as a seat belt are two of the most important risk factors for motor vehicle accidents and injuries.

Recommending buying a car with airbags (choice D) is not the most important discussion to have with this patient at this time. It is more important to encourage him to wear a seat belt in a car at all times. Crash victims who were wearing seat belts at the time of the accident have less severe injuries and are less likely to require hospital admission. Air bags prevent 18-19% of all automobile driver fatalities and 13% of right front passenger fatalities, over and above the fatality reduction with seat belt use alone. Air bags, however, are designed to be a supplemental restraint system to seat belts and are not sufficient on their own since they do not deploy in low-speed, rear, side, or rollover crashes.

It is not adequate to recommend buying a car with airbags and to wear a seat belt when driving (choice E). It is most important to encourage the patient to wear a seat belt at all times.

 

 

A 72-year-old widow comes to the office for a health maintenance examination. She has hypertension, type 2 diabetes, osteoporosis, and has suffered from 2 cerebral vascular accidents. She fell and broke her hip last year and underwent hip replacement surgery 6 months ago. She also suffers from chronic, neuropathic pain in her lower legs and feet. Her daily medications include enalapril, furosemide, alendronate, warfarin, glipizide, ibuprofen, and oxycodone (controlled release). She has been having a lot of trouble paying her bills and has been asking you for the past few years to discontinue some of her medications because they are “bankrupting” her. Her 29-year-old grandson recently moved into her apartment with her to help pay the rent. She tells you that he does not have a steady job, but she believes that he does odd jobs around the neighborhood, and always manages to come up with the money for food and rent. Her blood pressure is 130/80 mm Hg and pulse is 65/min. Her blood glucose level is 109 mg/dL. Her physical examination is unchanged from the previous year. You tell her that she seems to be doing well, but you cannot discontinue any of her medications at this time. As she steps down from the examination table, she states that her “neuropathy” is much worse and that she believes that you “need to increase the dose of time-released oxycodone from 20 mg a day to 80 mg a day”. The most appropriate next step is to

  A. discontinue the oxycodone and give her a prescription for the fentanyl patch
  B. explore the situation further to determine if there is more to this request
  C. increase the dose of time-released oxycodone from 20 mg a day to 80 mg a day
  D. refer her to a pain management specialist
  E. seek consultation with a neurologist
Explanation:

The correct answer is B. This patient has already told you that she is having money problems, and since time-released oxycodone has become a drug of abuse by adolescents and young adults, it seems that her grandson may be involved in this plan. Some elderly patients have realized that selling their prescription for time-released oxycodone can help them pay for the rest of their medications. The fact that her grandson has recently moved into her apartment, does not have a steady job, and yet always manages to come up with the money for rent is somewhat questionable. Also, the specific request for an 80 mg dose as opposed to a 20 mg dose should raise some concern. Not many elderly patients come in with specific dosage requests.

While a fentanyl patch (choice A) may be more difficult to abuse than oxycodone (that can be crushed and then snorted/injected to produce a rapid, intense high), it is important to first try to explore the reasons behind this patient’s request. Chronic pain is a serious problem that often requires narcotic medication; however you should become concerned when an elderly patient asks for a specific dosage increase of a narcotic that is known to be abused. In this case, she may be giving it her grandson who in turn sells it for a lot of money to addicts.

It is inappropriate to increase the dose of time-released oxycodone from 20 mg a day to 80 mg a day (choice C) without first trying to discuss the situation and determine if her grandson may somehow be involved in this request. Also, if you determine that an increase is necessary, it is probably appropriate to increase it to 40mg, not 80mg.

A pain management specialist (choice D) and a neurologist (choice E) may be necessary in the future, however at this time it is most appropriate to explore the situation further to determine if there is more to this request. It is possible that she is asking for this specific increase to give it to her grandson who will then sell the pills to help pay for food, rent, and her other medications.

 

 

A 22-year-old healthy nursing student comes to the clinic for her annual physical examination. She has no past medical history and her previous surgical history is remarkable for a dilation and curettage three years prior to pregnancy. She has a history of chicken pox as a child. She drinks 2-3 beers three times per week and denies cigarette smoking. She is not in a monogamous relationship and she is sexually active. Her last menstrual period was 3 days ago. Her vital signs and physical examination are unremarkable. Her vaccination schedule is as follows:

MMR: age 16; Td: age 16; PPD: negative 6 months prior; Hepatitis B titer: positive 1-year prior.

The most appropriate intervention at this time is to administer the

  A. Hepatitis B vaccine
  B. influenza vaccine
  C. pneumococcal vaccine
  D. Td vaccine
  E. varicella vaccine
Explanation:

The correct answer is B. The concept of a healthy visit came about around the turn of the century and has recently been called into question. Current recommendations are largely based on the USPSTF (United States Preventative Services Task Force) guidelines from 1996. The data behind these recommendations vary from excellent to non-existent. A flu shot is currently recommended in health care workers, patients with chronic disease, and in patients who ask for one. A “flu shot” is the influenza vaccine.

This patient has had a positive Hep B titer recently and therefore does not need a Hepatitis B vaccine (choice A).

The pneumococcal vaccine (choice C) is recommended for patients with chronic disease, patients over the age of 65, or institutionalized patients over the age of 50.

This patient does not require a Td vaccine (choice D) as she has had a booster within the previous eight years.

The varicella vaccine (choice E) is recommended in day-care workers or teachers or in-patients with no history of varicella.

 

 

A 72-year-old widow comes to the office for a health maintenance examination. She has hypertension, type 2 diabetes, osteoporosis, and has suffered from 2 cerebral vascular accidents. She fell and broke her hip last year and underwent hip replacement surgery 6 months ago. She also suffers from chronic, neuropathic pain in her lower legs and feet. Her daily medications include enalapril, furosemide, alendronate, warfarin, glipizide, ibuprofen, and oxycodone (controlled release). She has been having a lot of trouble paying her bills and has been asking you for the past few years to discontinue some of her medications because they are “bankrupting” her. Her 29-year-old grandson recently moved into her apartment with her to help pay the rent. She tells you that he does not have a steady job, but she believes that he does odd jobs around the neighborhood, and always manages to come up with the money for food and rent. Her blood pressure is 130/80 mm Hg and pulse is 65/min. Her blood glucose level is 109 mg/dL. Her physical examination is unchanged from the previous year. You tell her that she seems to be doing well, but you cannot discontinue any of her medications at this time. As she steps down from the examination table, she states that her “neuropathy” is much worse and that she believes that you “need to increase the dose of time-released oxycodone from 20 mg a day to 80 mg a day”. The most appropriate next step is to

  A. discontinue the oxycodone and give her a prescription for the fentanyl patch
  B. explore the situation further to determine if there is more to this request
  C. increase the dose of time-released oxycodone from 20 mg a day to 80 mg a day
  D. refer her to a pain management specialist
  E. seek consultation with a neurologist
Explanation:

The correct answer is B. This patient has already told you that she is having money problems, and since time-released oxycodone has become a drug of abuse by adolescents and young adults, it seems that her grandson may be involved in this plan. Some elderly patients have realized that selling their prescription for time-released oxycodone can help them pay for the rest of their medications. The fact that her grandson has recently moved into her apartment, does not have a steady job, and yet always manages to come up with the money for rent is somewhat questionable. Also, the specific request for an 80 mg dose as opposed to a 20 mg dose should raise some concern. Not many elderly patients come in with specific dosage requests.

While a fentanyl patch (choice A) may be more difficult to abuse than oxycodone (that can be crushed and then snorted/injected to produce a rapid, intense high), it is important to first try to explore the reasons behind this patient’s request. Chronic pain is a serious problem that often requires narcotic medication; however you should become concerned when an elderly patient asks for a specific dosage increase of a narcotic that is known to be abused. In this case, she may be giving it her grandson who in turn sells it for a lot of money to addicts.

It is inappropriate to increase the dose of time-released oxycodone from 20 mg a day to 80 mg a day (choice C) without first trying to discuss the situation and determine if her grandson may somehow be involved in this request. Also, if you determine that an increase is necessary, it is probably appropriate to increase it to 40mg, not 80mg.

A pain management specialist (choice D) and a neurologist (choice E) may be necessary in the future, however at this time it is most appropriate to explore the situation further to determine if there is more to this request. It is possible that she is asking for this specific increase to give it to her grandson who will then sell the pills to help pay for food, rent, and her other medications.

 

 

A 5-month-old girl is brought into the office for the first time. Her mother tells you that her child has not received the 4-month vaccinations yet because the day after her 2-month shots she developed a “high fever.” The mother is now concerned about her daughter receiving “shots.” After explaining the risk and benefits of vaccinations, you should

  A. administer all the recommended vaccines except for the DTaP vaccine if the mother agrees
  B. administer all the recommended vaccines except for the pertussis part of the DTaP vaccine if the mother agrees
  C. agree with the mother and delay the immunizations until the child gets older
  D. give all of the recommended vaccinations if the mother agrees
  E. give the OPV instead of the IPV if the mother agrees
Explanation:

The correct answer is D. The most common immunization, which causes fever is the DTaP. The other vaccinations usually given at the 2-month visit are IPV, Hepatitis B, Hib, and now the pneumococcal vaccine. The DTaP, now given, is the acellular preparation. It has fewer side effects then the DTP (which is the whole cell preparation). There are a few true contraindications for the administration of immunizations and they are an anaphylactic reaction to a vaccine, moderate or severe illness (shots should be resceduled), convulsions, hypotonic hypotensive state, or encephalopathy within 48 hours of any vaccine. If she had a moderate or severe illness, the shots should be rescheduled when the illness resolves. Since fever after immunizations isn’t a contraindication, all immunizations should be given at this time.

Fever after immunization is not a contraindication for receiving DTaP in the future (choice A).

Although the pertussis is usually the portion of the vaccine that causes the fever, the benefits of receiving the immunization outweigh the risk of getting a fever. Therefore it is inappropriate to administer all the recommended vaccines except for the pertussis part of the DTaP vaccine (choice B).

Agreeing with the mother (choice C) is incorrect because of the incredible benefits of immunizations. It has helped decrease childhood mortality remarkably. So as long as the child doesn’t have a true contraindication, the shots should not be delayed

There is never a reason to give OPV instead of IPV (choice E).

 

A 50-year-old woman is admitted to the hospital with right upper quadrant pain. She states that in the past she has had occasional bouts of pain in the same area, particularly after fatty meals. She denies any vomiting, but does feel nauseated and says that there has been no diarrhea. She has no past medical history and has not seen a doctor since childhood. She smokes 2 packs of cigarettes a day and drinks 2 glasses of wine with dinner. Her temperature is 37.8 C (100 F) and blood pressure is 130/88 mm Hg. Her abdomen is soft with normal bowel sounds, and she has a positive Murphy’s sign. Laboratory studies reveal a leukocyte count of 15,600mm3 with 90% segmented neutrophils, AST 120 U/L, ALT 150 U/L, alkaline phosphatase 120 U/L, bilirubin (total) 0.9 mg/dL, and amylase 66 U/L. She is diagnosed with acute cholecystitis and undergoes laparoscopic cholecystectomy. Before discharge, she tells you that she likes your bedside manner and would like to follow up with you in the clinic. You should tell her that when she comes to see you, she will need a periodic health maintenance examination which will include a

  A. mammogram every 5 years to screen for breast cancer
  B. mammogram every 2 years to screen for breast cancer
  C. pelvic ultrasound bi-annually to screen for ovarian carcinoma
  D. yearly chest x-ray for lung cancer screening
  E. yearly mammogram and yearly fecal occult blood testing for colon cancer
Explanation:

The correct answer is E. It is important to identify patients admitted to the hospital for an acute illness who have not been followed in the medical system and had adequate health care screening. When they return for follow up appointments, this can be instituted. In a 50-year-old woman, yearly screening with mammography for early breast cancer detection and fecal occult blood screening for early colon cancer detection are currently recommended. Flexible sigmoidoscopy every 2-3 years is also included among many of the current recommendation guidelines to screen for colonic malignancy.

As mentioned above, a mammogram every 5 years (choice A) or every two years (choice B) is too infrequent..

There are currently no recommended screening procedures for ovarian carcinoma (choice C). Evaluation for this disease should be limited to those patients with some predisposing factors or symptomatology.

Yearly chest x-ray for lung cancer screening (choice D) is not currently recommended. Early detection with low dose CT scanning is currently under heavy investigation and shows much promise.

 

 

A 5-year-old boy is brought to the office by his mother for a routine physical examination before beginning a new school. The family recently moved to your town and this is the first time you are meeting them. The mother tells you that the child was born at term with no complications and has been healthy ever since. She tells you that she “does not believe in vaccines” and none of her 3 children have received any vaccinations. You try to explain the benefits and risks of immunizations and listen to her concerns. However, she continues to be adamant that her “children will remain unimmunized.” You give her a vaccine information statement and other printed material about vaccinations. The child appears happy and healthy and a complete physical examination is normal. The most appropriate next step is to

  A. call the child’s new school and inform them of the child’s vaccination status
  B. contact the child protective services agency to report child neglect
  C. document your discussion about vaccines in the patient’s chart
  D. inform the local board of health of the mother’s decision
  E. tell her that her decisions are foolish and put her children at great risk
Explanation:

The correct answer is C. In practice, you will come in contact with individuals with misconceptions about vaccines. Web sites, news programs, and other groups often put out misinformation with exaggerated side effects. It is important to discuss the benefits and risks of immunizations and to provide trustworthy material about vaccines. If you are unable to change the person’s mind, you need to document your discussion in the patient’s chart to reduce liability if this patient were to contract a vaccine-preventable disease.

It is inappropriate to call the child’s new school and inform them of the child’s vaccination status (choice A). Schools often require a vaccination history prior to entry so they will most likely be aware of the situation soon.

You should not contact the child protective services agency to report child neglect (choice B). Neglect is when a caretaker fails to provide adequate nutrition, supervision, or medical care for a child. Even though it seems neglectful to withhold vaccinations from a child, it is not something that has to be reported as neglect.

While it seems like it might be important to inform the local board of health of the mother’s decision (choice D), it is not appropriate. The child’s school will soon find out that the child is unimmunized and decide whether or not to allow entry. As the physician, you should try to persuade her to vaccinate her children, and if she refuses, you should document it in the chart. The local board of health is where you report notifiable diseases, such as gonorrhea and syphilis.

Since you want to establish a good physician-patient relationship with this family with hopes of changing her mind about vaccines, it is wrong to insult her by telling her that her decisions are foolish and are putting her children at great risk (choice E). You should explain the benefits and risks of immunizations and try to persuade her that vaccinations are routinely given with few side effects. You should not insult her or else she may not come back. This might sound good, but you want to provide the best care and going forward you want to try and help her understand the truth about vaccines.

  A 24-year-old woman presents to your clinic for a routine visit. She has been your patient for two years and is a graduate student at the local university. She has no past medical history and takes only oral contraceptive pills daily. Her review of systems is largely unremarkable with only some complaints of mild fatigue and stress, which she related to her upcoming dissertation defense. Her last visit to you was 11 months ago and a perusal of her lab data and chart from a previous visit reveals nothing out of the ordinary. She smokes one pack of cigarettes per week and is monogamous with her boyfriend of three years. Her blood pressure is 112/80 mm Hg, heart rate is 74/min, and temperature is 36.7 C (98.0 F). Her physical examination is unremarkable. Her health care plan has been sending her literature urging her to get “appropriate screening” so she is concerned that she is not receiving age-appropriate screening. The most appropriate next step in management is

  A. hematocrit
  B. PPD skin testing
  C. Pap smear
  D. rapid plasma reagin test
  E. urinalysis
Explanation:

The correct answer is C. The concept of a healthy visit came about around the turn of the century and has recently been called into question. Current recommendations are largely based on the USPSTF (United States Preventative Services Task Force) guidelines from 1996. The data behind these recommendations vary from excellent to non-existent. For a female, a Pap smear is recommended starting at age 18 or after her first intercourse and every one to three years thereafter.

There is no indication to test hematocrit (choice A) unless the patient is pregnant or has significant signs and symptoms of anemia.

PPD skin testing (choice B) is reserved for health care workers, alcoholics, IV drug abusers, diabetics, or end-stage renal patients.

The RPR or rapid plasma reagin test (choice D) is used to test for syphilis and is recommended if the patient is pregnant, has a history of sexual transmitted diseases, or of she engages in sexual activity for money.

A screening urinalysis (choice E) is reserved for when a female of this age presents with signs or symptoms suggestive of a urinary tract infection. These would include fever, flank pain, and dysuria.

 

 

  A 59-year-old woman comes to the office for a periodic health maintenance examination. She has no particular complaints at this time but she is very concerned about having a stroke. Her best friend’s husband recently suffered a cerebrovascular accident that left him with residual paralysis and she is determined to prevent this from happening to her and her husband. She is moderately obese, has diabetes mellitus and hypertension, smokes a pack of cigarettes a day, and has elevated blood cholesterol. She should be told that the most significant way for her to reduce her risk of stroke is to

  A. lose weight
  B. maintain tight glucose control
  C. quit smoking
  D. reduce blood cholesterol
  E. treat high blood pressure
Explanation:

The correct answer is E. Cerebrovascular accidents (or strokes) are one of the most common causes of death in the United States. The risk factors for a stroke are hypertension, diabetes mellitus, cigarette smoking, elevated cholesterol, older age, obesity, and a family history of stroke. There are many medical interventions and lifestyle modifications that have been shown to decrease the risk of suffering a stroke. But the single most significant risk factor is hypertension, so this patient should be told that the most significant way for her to reduce her risk of ischemic stroke is to treat her high blood pressure.

All of the other choices, lose weight (choice A), maintain tight glucose control (choice B), quit smoking (choice C), and reduce blood cholesterol (choice D), should all be encouraged to reduce her risk of a stroke. But as stated above, hypertension is the single most important risk factor for stroke.

 

 

A 56-year-old man comes to the office for a periodic physical examination. His has not been seen in the office in 2 years and has no specific complaints, however, he is a bit concerned about screening tests that he may be missing. A review of systems reveals no suspicious signs or symptoms. His family history is remarkable for a father that died of heart disease at age 60 and a mother with diabetes mellitus. The patient has recently had a cholesterol screening test at work and both his total cholesterol and HDL/LDL fractions were within acceptable limits. The patient reports that he has never had any surgical procedures or diagnostic procedures. The most appropriate screening for this patient is

  A. colonoscopy every 7-10 years
  B. exercise stress testing annually
  C. glucose tolerance testing every 3-5 years
  D. prostate specific antigen levels annually
  E. sigmoidoscopy every 7-10 years
Explanation:

The correct answer is A. Screening tests are those tests that have been demonstrated to have both cost effectiveness and efficacy in the detection of a specific disease. Guidelines for screening tests are based both upon age and risk factors. For men over the age of 55, a colonoscopy is recommended as superior to both digital rectal exam with heme testing and sigmoidoscopy for the early detection of colonic cancer and polyps. Once done, the repeat is suggested every 7-10 years.

Despite the age of this patient and a positive family history of cardiac disease (2 risk factors), there are no routine indications for screening exercise stress testing (choice B). The false positive rate is too high leading to more invasive procedures that patients do not actually need.

Screening tests for diabetes in the general population consist of blood glucose levels. Oral glucose tolerance testing (choice C) is used to test pregnant women for gestational diabetes. In adults, 2 consecutive blood glucose levels of 126 mg/dL or greater is diagnostic of diabetes.

Despite the prevalence of prostate cancer in men over the age of 50, routine prostate specific antigen levels (choice D) are not recommended. Elevations do not signify cancer (often just benign hypertrophy) and many cancer patients have normal PSA levels.

Since sigmoidoscopy (choice E) reaches only to the sigmoid colon, it will miss the more distal lesions. Recent studies have indicated that this failure of detection rate can approach 10-15%. For this reason, colonoscopy has become the preferred screening method for colonic lesions by many physicians. But if it were to be used for screening, it should be done every 3-5 years and should possibly be combined with annual fecal occult blood testing.

 

A 4-year-old boy is brought to the clinic for a well-child examination and immunizations. He is usually very healthy, but he happens to have a “cold and fever” for the past 3 days. The mother reports that he is doing well at home and in school. You notice in his chart that he has missed a few scheduled appointments in the past but he has had no serious problems. His temperature is 38.0 C (100.0 F). Physical examination shows clear nasal discharge, but is otherwise unremarkable. He is due for the measles mumps rubella, polio, and diphtheria tetanus and pertussis vaccines at this time. The most appropriate next step in management is to

  A. administer all of the vaccines at this time
  B. administer the inactivated polio and the diphtheria tetanus and pertussis vaccines only at this time
  C. advise him to return for the vaccines when he is feeling better
  D. give him the vaccines at the next scheduled well-child examination
  E. prescribe azithromycin and schedule an appointment in 10 days for immunizations
Explanation:

The correct answer is A. A minor illness with or without fever is not a contraindication for immunizations. There are often misconceptions about this idea. According to the American Academy of Pediatrics and the Report of the Committee on Infectious Diseases, there is no evidence that indicates that immunizations should not be given during a minor febrile illness. This is important because deferring immunizations to a later date may lead to inadequately immunized children. This is especially important in this case because he has missed regularly scheduled appointments in the past.

All vaccines can be administered at this visit, including the MMR (which is a live vaccine), even though he has a minor febrile illness. A minor respiratory illness with fever is not a contraindication for live vaccines because these children have similar serologic responses to vaccines as afebrile, well children. Therefore, it is incorrect to administer the inactivated polio and the diphtheria tetanus and pertussis vaccines only at this time (choice B).

Since he has missed scheduled appointments in the past and has a minor febrile illness, this is not a contraindication for immunizations. It is incorrect to advise him to return for the vaccines when he is feeling better (choice C). If you advise him to return when he is feeling better, he may not return and this will lead to an inadequately immunized child.

It is incorrect to give him the vaccines at the next scheduled well-child examination (choice D). He has missed appointments in the past and there is no reason that he should not be given the vaccines that he is due for at this time.

Azithromycin (choice E) is not given for 3 days of “cold and fever” and clear nasal discharge. It is given for infections due to Moraxella catarrhalis and Legionella pneumophila, not for viral infections, which he most likely has. Also, he should be given the immunizations now, and not in 10 days.

 

 

A 23-year-old woman who works as a waitress comes to the office because she got married 6 months prior, and is now interested in trying to conceive. She is hypothyroid and takes levothyroxine once a day, but is otherwise in good health. She runs approximately 7 miles, 5 days a week, does not smoke cigarettes, drinks wine on special occasions, and does not take any illegal drugs. She is currently on a combined oral contraceptive that she has been taking for the past 6 years. Her physical examination is entirely normal. In addition to taking her off of her oral contraceptive, the most appropriate counseling for her at this point is

  A. she should also stop her levothyroxine, as it can cause severe birth defects
  B. she should begin taking a multivitamin and folic acid and stop drinking wine
  C. she should cut down on her exercise routine because so much impact exercise could jeopardize implantation
  D. she should expect that her baby will be small for gestational age with a high chance of thyroid dysfunction as well
  E. she should not expect to get pregnant for at least 6 months, as that is one of the side effects of long term oral contraceptive use
Explanation:

The correct answer is B. This patient is a normal, healthy woman who needs preconceptual counseling. Important points in preconceptual counseling are correct nutritional supplementation for organogenesis in addition to limited alcohol intake to decrease fetal alcohol syndrome. A multivitamin in addition to 4 mg of folic acid are recommended for all women trying to conceive without risk factors.

She should also stop her levothyroxine, as it can cause severe birth defects (choice A) is incorrect. There is no evidence of teratogenesis of levothyroxine, as it is in pregnancy category A. Many hypothyroid women conceive while taking this drug.

Impact exercise does not jeopardize implantation. She will need to cut down on this exercise regimen once her pregnancy is established, keeping her heart rate at or below 140 beats per minute. However, preconceptually, she does not need to cut down on her exercise routine (choice C).

In general, infants of hypothyroid mothers appear healthy and without evidence of thyroid dysfunction. She should not expect any abnormalities (choice D).

She should not expect to get pregnant for at least 6 months, as that is one of the side effects of long-term oral contraceptive use (choice E) is incorrect. Once oral contraceptives are stopped, a patient can expect to get pregnant at any time. There is a rapid return to fertility with average delay in ovulation of 1-2 weeks.

 

 

A 27-year-old woman comes to the office for a periodic health maintenance examination. She is healthy and has no specific complaints. She works at an advertising agency, has a “live-in” boyfriend, exercises a few times a week, eats a low-fat diet, and rarely drinks alcohol. She smokes a pack of cigarettes every 3 days. Her Pap smears have always been normal. The last one was 2 years ago. She currently uses condoms for birth control, but lately her boyfriend has been “nagging” her to “go on the pill.” She is 157 cm (5 ft 2 in) tall and weighs 54 kg (119 lb). Her blood pressure is 120/80 mm Hg and pulse is 60/min. Physical and pelvic examinations are unremarkable. In discussing oral contraceptive pills (OCPs) with this patient, you should inform her that:

  A. Condoms do not need to be used to prevent chlamydial infections
  B. Her menstrual periods will become longer and heavier
  C. She must stop smoking before you prescribe oral contraceptive pills
  D. Oral contraceptive pills decrease the risk for gonococcal pelvic inflammatory disease
  E. Oral contraceptive pills increase the risk for ovarian cancer
Explanation:

The correct answer is D. OCPs have many beneficial effects, and one is that they decrease the risk for gonococcal PID. This is thought to be due to the effects that OCPs have on the endometrium and cervix leading to increased endocervical mucus that resists the spread of gonococcal infection from the cervix to the endometrium and then the fallopian tubes.

OCPs have been associated with an increased risk of chlamydial infections and therefore, condoms SHOULD be used to prevent chlamydial infections (choice A). This increased risk may be associated with the fact that OCPs widen the ectocervix , increasing the area of mucus-secreting cells and making the individual more susceptible to chlamydial infections. It seem that this increases the risk of chlamydial infections and decreases the risk for gonococcal infections

It is incorrect to inform her that her menstrual periods will become longer and heavier (choice B) because OCPs are typically associated with shorter, less painful, and scanty menses.

Smoking in women that are over the age of 35 is a relative contraindication to the use of OCPs. While smoking may increase the risk of complications in women under 35, it is not a contraindication to their use. You should advise her that it is best if she quits (in general and because of the OCPs), but it is not correct to inform her that she must stop smoking before you prescribe oral contraceptive pills (choice C).

Oral contraceptive pills have been associated with a decreased risk for ovarian cancer, not an increased risk (choice E).

 

 

A 41-year-old woman comes to your clinic because she injured her arm during a fall 2 days ago. She has had multiple visits to the clinic for various reasons, with a significant number of visits over the past two years for fractures and falls. The patient has consistently reported that her falls and fractures are related to her job as a cleaning woman at a local factory. Her most recent visit three weeks ago was for a bruised face requiring six sutures for a laceration over her forehead. The patient lives with her boyfriend of 15 years. He is unemployed and has been hospitalized for alcohol withdrawal seizures three times. The patient reports that she and her boyfriend argue fairly often but that he is often drunk and does not remember what he says or does. The patient has a 7th grade education and a 20-year smoking history. She has four children all from her current partner. She reports that her children also fall quite a bit and have been treated for fractures and bruises as well. When you inquire about her home life, the patient offers no response and tries to leave the clinic. The most accurate statement concerning screening for domestic violence is:

  A. Do not interview the patient alone
  B. A few direct questions about abuse may be recommended in adult patients
  C. Routine screening is indicated only for high-risk patients
  D. Routine screening is not indicated
  E. Women are more likely to report domestic violence to a non-medical person
Explanation:

The correct answer is B. According to the United States Preventative Services Task Force, including a few direct questions about abuse may be recommended in adult patients. Because of the high prevalence of domestic violence (DV) and the wide range of clinical settings in which it can be seen, a few direct questions may be a part of preventative care. Because of this prevalence, the belief that routine screening is not indicated (choice D) is simply not true. If screening is limited to high risk patients only (choice C), then a substantial number of DV cases would be missed as the “risk factors” for DV are loose and not clearly validated in the same way as risk factors for cardiovascular disease have been. The belief that women are more likely to report domestic violence to a non-medical person (choice E) is false. A number of studies have shown that if a physician takes the time to address the issue of possible DV, the woman is more likely to report it to them than to a non-physician. It is a key component of the interview that the patient be made to feel comfortable and interviewed alone (choice A).

 

 

You are the physician at a walk-in clinic for university students. A 19-year-old woman comes to the clinic stating that she has recently become sexually active and is thinking about starting oral contraceptives. She has no significant medical history and takes no medications. She exercises regularly and drinks alcohol on occasion. She has never smoked tobacco and denies any drug use. She is somewhat hesitant about taking hormonal drugs, but wants reliable protection against pregnancy. In addition, she heard that the pill can lead to embolism. In talking with her about thromboembolism in oral contraceptive users, it is most appropriate to include:

  A. Alcohol use may increase the risk of venous thromboembolism
  B. Family history of clotting disorders does not influence the risk of a venous thromboemboembolism while on oral contraceptives
  C. It is the estrogen component of the combined oral contraceptive that causes increased risk for a venous thromboembolism, progestin has no effect
  D. The risk of a venous thromboembolism is greater with pill use than it is in pregnancy
  E. The risk of a venous thromboembolism is the same with all of the available oral contraceptives
Explanation:

The correct answer is C. Venous thromboembolism is one of the complications of combined oral contraceptive use. It is the dose of the estrogen component alone that causes this increased risk, from baseline of approximately 4/100,000 to approximately 10/100,000. For example, 50 microgram pills have greater risk than 20-35 microgram pills. The progestin component does not have an effect on thromboembolism.

Tobacco use increases the risk for thromboembolic events. Alcohol use (choice A) does not seem to increase the risk.

Family history of clotting disorders such as factor V Leiden or protein C and S would alert the physician to test the patient for these disorders. If such clotting disorders are present in the patient, the use of combined oral contraceptives (choice B) increases her risk for a thromboembolic event.

The risk of venous thromboembolism in pregnancy is approximately 60/100,000 women per year, significantly greater than the risk for thromboembolism in oral contraceptive users (choice D).

Oral contraceptives are available in different doses of estrogen. Therefore, as thromboembolic events are increased with a higher estrogen component, all pills are not equal in risk (choice E).

 

 

A 16-year-old girl is brought to see you by her mother, who is a nurse in your office. The patient is hoping to play for the school volleyball team and needs a clearance form signed by a doctor. Her past medical history includes exercise-induced asthma and allergic rhinitis for which she takes albuterol and loratadine.During the interview with the patient, while you are speaking with her alone, she reveals that she has been sexually active with her boyfriend for the last 2 years. She has not yet discussed this fact with her parents. Her physical examination, including pelvic exam, is normal. According to the U.S. Preventive Services Task Force, at this time the patient should be screened for

  A. chlamydia
  B. gonorrhea
  C. hepatitis B
  D. HIV
  E. herpes simplex
  F. syphilis
Explanation:

The correct answer is A. In the 2001 update of their screening recommendations, the U.S. Preventive Services Task Force strongly recommended routine screening for chlamydia in all sexually active women aged 25 and younger, as well as in asymptomatic women older than 25 who are at high risk.

Other sexually transmitted infections, such as gonorrhea (choice B), hepatitis B (choice C), HIV (choice D), herpes simplex(choice E), and syphilis (choice F) may also be considered if the patient had specific complaints related to these conditions. However, in an asymptomatic person, it is not recommended to routinely screen for these conditions.

 

 

A 24-year-old nurse returns to the clinic for a follow up of her annual physical examination. Two days ago, she presented to the clinic and had a PPD test placed in her right forearm. She is an otherwise healthy woman who has been a nurse for 2 years. She has been working at the same inner city hospital during nursing school and since graduation. Her nursing duties consist of care for medical patients, mostly on the general medical wards. She states that these patients are often homeless, and many are of very low socioeconomic status. On examination of the right forearm, she has a 12-mm induration at the site of the PPD placement. She is worried that the she has tuberculosis. At this time the most correct statement about her condition is:

  A. A chest radiograph x-ray should be obtained before any further treatment is initiated
  B. No additional studies are required because a 15-mm induration is considered a positive test
  C. She is a high-risk patient and therefore cannot receive isoniazid therapy for latent infection
  D. The patient has latent infection and treatment should be initiated immediately
  E. This patient has a latent infection but no therapy is indicated
Explanation:

The correct answer is A. The patient has a positive PPD test. The PPD is an intradermal injection of 0.1 ml of purified protein derivative containing 5 tuberculin units. Results are read 48-72 hours after placement. For health care workers, an induration of greater than 10mm is considered positive. Before the treatment for latent infection with isoniazid can be started, a chest radiograph excluding active infection must be obtained.

A 15 mm induration (choice B) is considered positive in persons with no identifiable risk factors for Tb. Since screening of the general population is not indicated, most persons screened will never display this type of induration. A 10-mm induration is considered positive for a health care worker.

A high-risk patient (choice C) is generally those who are over age 35. These persons are at excessive risk for isoniazid induced hepatitis.

This patient does have latent infection and if her chest radiograph shows no active disease, isoniazid therapy is indicated (choice D). But a chest x-ray needs to be performed before initiating therapy.

As discussed in choice A, prior to starting preventative therapy for latent infection, the patient must have active disease excluded (choice E).

 

 

A 30-year-old woman comes to the office for a periodic health maintenance examination. She has no complaints at this time. She works as a sales manager of a department store, goes to an aerobic exercise class after work, 4 times a week, drinks a glass of wine every 3-5 days, and does not smoke cigarettes. She tells you she recently stopped taking her oral contraceptive pills because she and her husband want to start trying to have a baby soon. Her menstrual periods come at regular 28-day intervals and typically last for 6 days. Her last menstrual period was 10 days ago. Her blood pressure is 110/70 mm Hg and pulse is 60/min. Physical examination is unremarkable. A pelvic examination is unremarkable. You decide to perform a Pap smear because you see that her last one was 2 years ago and was normal, as always. The most appropriate next step is to

  A. do a urine pregnancy test
  B. obtain chlamydial and gonorrheal cultures
  C. perform a clinical breast examination
  D. recommend a daily multivitamin with folic acid
  E. test her for hepatitis B surface antigen
Explanation:

The correct answer is D. The United Stated Preventive Services Task Force (USPSTF) recommends that all women planning to become pregnant take a daily multivitamin supplement containing folic acid. It is recommended that they start taking it at least 1 month prior to conception and continue through the first trimester to reduce the risk of neural tube defects. According to the USPSTF, taking a daily multivitamin containing folic acid is also recommended for all women capable of becoming pregnant. This is to reduce the risk of neural tube defects in unplanned pregnancies.

A urine pregnancy test (choice A) is unnecessary at time because this patient’s last menstrual period was only 10 days ago and she has not missed a period. It is very unlikely that she is pregnant at this time.

It is unnecessary to obtain chlamydial and gonorrheal cultures (choice B) because this patient is asymptomatic and is not considered at high risk for these infections. The USPSTF recommends that the routine screening of asymptomatic persons for these infections should typically be limited to sexually active adolescents and people that fall into high-risk groups, including those with a prior sexually transmitted disease, multiple partners, inconsistent barrier contraception usage, being unmarried, and being under 25 years of age. High-risk pregnant women should be screened. They say that the routine screening of asymptomatic adults for chlamydia and gonorrhea is not recommended.

The USPSTF concludes that there is insufficient evidence to recommend for or against routine clinical breast examinations (choice C) alone to screen for breast cancer. They recommend screening mammography, with or without clinical breast examination, every 1–2 years for women aged 40 and older. According to these guidelines, this asymptomatic 30-year-old woman does not need a CBE at this time.

The USPSTF recommends that pregnant women be tested for hepatitis B surface antigen (choice E) at their first prenatal visit. Since she is most likely not pregnant yet (her last menstrual period was 10 days ago), this test is unnecessary at this time.

 

 

A 3-year-old boy is brought to the office for a routine examination. The mother tells you that he is a “great kid,” always laughing and playing with other boys in the neighborhood. She says that he is very excited because they are currently having a pool put into their backyard that will be ready just in time for summer. She is a bit concerned about his safety, and asks if you know anything about childhood accidents and pool safety. You should advise her that

  A. Adult supervision will eliminate almost all accidents
  B. A fence should be installed around the pool
  C. He needs to wear a life jacket when he is near the pool
  D. Pool covers will eliminate all accidents
  E. She must enroll him in a swim class to teach him to swim
Explanation:

The correct answer is B. Fencing outdoor pools is the most effective way to safeguard children against accidental drowning. Children may wander into the pool when nobody is around, and by the time somebody goes to look for him, it may be too late. A fence will help prevent this type of accident.

It is inappropriate to tell her that adult supervision will eliminate almost all accidents (choice A) because more than half of all drownings occur when a child is supposedly under the supervision of parents or adults. Children have a tendency to wander off, and the best way to protect them from drowning is to have a fence around the pool so that they cannot get near it.

While a life jacket when he is near the pool (choice C) may seem like a good idea, it requires that an adult put it on him when they know that he is planning to go near the pool. Accidents may occur when nobody else knows that he is approaching the pool, and therefore, only a fence can ensure that he does not approach the pool unattended.

A pool cover (choice D) may be helpful if it is always used. However, the child can still slip through the side and nobody will know that they are underneath. Also, it is possible to drown in a small amount of water that collects on the surface. A fence around the pool (possibly in addition to a cover) will help to prevent these situations.

Enrolling him in a swim class (choice E) may be a good idea. However, it may not improve survival after submersion. Hyponatremia and water intoxication have occurred in toddlers when they have been unable to prevent submersion. A fence is the most effective way to prevent accidental drowning.